Vous êtes sur la page 1sur 28

MINISTRY OF PUBLIC HEALTH OF UKRAINE

Department of human resources policy, education and science

Testing Board

Student ID Last name

Variant ___________________

Test items for licensing examination

Krok 2
MEDICINE
General Instruction
Every one of these numbered questions or unfinished statements in
this chapter corresponds to answers or statements endings. Choose
the answer (finished statements) that fits best and fill in the circle
with the corresponding Latin letter on the answer sheet.

Authors of items: Abaturov O.E., Agarkov V.I., Alexeyenko L.I., Andriyetc O.A., Andrusyak O.V.,
Bagiryan I.O., Baralo I.V., Baranova N.P., Barcikhovskyi A.I., Belyakova V.I., Berbetc A.M.,
Ber’ozov V.M., Bobcovich K.O., Bobrischev K.A., Boichuk A.V., Budayeva I.V., Butvin I.M.,
Caralunga V.M., Chabanenko I.P., Cherhyavska N.B., Chuyko A.P., Delva YU.V., Denysenko N.M.,
Derkach V.G., Dobrovolska L.M., Dobryanskyi D.O., Docenko T.M., Dorofyeyev A.E., Doroschenko T.V.,
Drupp YU.G., Dyacuk A.M., Dzuba G.A., Fedorenko O.YE., Fundyur N.M., Galicheva N.O.,
Galushchenko S.A., Genyk N.I., Gerasymenko O.I., Grechko S.I., Grydasova V.D., Gryn N.V.,
Gryshchenko V.I., Kablukova O.K., Kalens’kyi B.KH., Kaliy V.V., Kalus’kyi Z.V., Kalsada I.N.,
Kaporina N.V., Karachevskyi A.B., Karliychuk O.O., Kokalko M.M., Kolomiyec V.V., Koloskova O.K.,
Komisarenko YU.I., Kondratenko P.G., Kondratyuk V.A., Kondrat’yev V.O., Konopkina L.I.,
Konopl’ova L.F., Kopyov YU.O., Kotcuta G.I., Kovalenko S.V., Koval’ova L.M., Kovtunenko R.V.,
Krasnopolska I.I., Kravchenko O.V., Kravec O.V., Krut YU.YA., Kryachkova L.V., Krysa V.M.,
Kubyschkin V.F., Kudiyevskyi A.V., Kudrevych O.M., Kuschnirenko A.G., Lacusta N.M., Latypova G.A.,
Lipnyckyi T.M., Logachova T.V., Loginova I.O., Loscutova I.E., Makyeyev S.I., Malanchuk L.M.,
Maruschko YU.V., Matcynin O.M., Medvedchuk G.YA., Melnykov S.M., Melnychuk L.V.,
Mirgorodskyi D.S., Mischchenko V.P., Mityunina N.I., Mizyuk M.I., Mostovyi YU.M., Oliynyk O.YE.,
Orel YU.G., Pakholchuk T.M., Pavlova N.O., Pepenin V.R., Perepichka M.P., Pokudko M.I., Polyak S.D.,
Polyakov A.E., Ptyimak S.G., Prokhorov YE.V., Prokhorova M.P., Prunchak I.F., Prus L.O., Pukhalska N.S.,
P’yatnyckyi YU.S., Rak L.M., Rusanova L.A., Ryabenko E.B., Sabadasch V.YE., Sakhaltyuev A.D.,
Sander S.V., Savchak V.I., Sels’ka O.V., Schcherbatyuk N.YU., Schcherbinin O.V., Schevchenko O.A.,
Schevchenko R.S., Schkvarkovs’kyi I.V., Schkurba A.V., Schorikov YE.I., Schvygar L.V., Sichanova O.V.,
Skiban V.O., Sladkova L.M., Smirnova V.L., Smyrnova O.V., Smolyak L.L., Snizhko S.S., Solodova I.V.,
Sotnik YU.P., Sukharyeva I.A., Suslyk Z.B., Svyrydova V.V., Synoverska O.B., Sychova V.V.,
Talalayenko I.O., Tarallo V.L., Titarenko O.V., Tkachenko L.O., Tokarenko I.I., Tomash O.V.,
Tomkiv V.M., Tyuyeva N.V., Uschenina N.S., Vasiluk V.M., Vasil’yeva N.V., Velika N.V.,
Vinentcov YU.O., Vitovska O.P., Voloschin O.M., Volyans’ka A.G., Vorkhlyk M.I., Yakovenko I.K.,
Yaremenko O.B., Yatcenko N.G., Yurchenko I.V., Zaporozhec V.K., Zayarskyi M.I., Zaycev V.I.,
Zheliba M.D., Zhuchenko L.M., Zhuk S.I., Zhuravel V.I., Znakhurenko L.S. and Committees of professional
expertise.

Item reviewers. Abaturov O.E., Anisimov E.M., Borisova T.P., Berezov V.M., Bryukhanova S.T.,
Chuyko A.P., Chursina T.YA., Dindar O.A., Gubka O.V., Gucalenko O.O., Istomin A.G., Kanikovsky O.E.,
Kaplina L.E., Karapetyan K.G., Kobec T.V., Kolesnik O.M., Kolosovich I.V., Kondratenko P.G.,
Kravchenko O.V., Kuzmina I.YU., Kutovoy O.B., Litvynenko N.G., Lugovskov O.D., Masik O.M.,
Mikhaylov V.V., Mityunina N.I., Orel YU.G., Prishlyak O.YA., Prokhorova M.P., Safonova O.V.,
Shekhovcova T.G., Sidorova L.L., Smolyak L.L., Solovyeva G.A., Sukhoplueva N.I., Tkachuk O.L.,
Vakaluk I.P.

The book includes test items for use at licensing integrated examination “Krok 2. Medicine” and
further use in teaching.

The book has been developed for students of medical, pediatric and medical-and-prophylactic
faculties and academic staff of higher medical educational establishments.

Approved by Ministry of Public Health of Ukraine as examination and teaching


publication based on expert conclusions (Orders of MPH of Ukraine of
14.08.1998 №251, of 27.12.1999 №303, of 16.10.2002 №374, of 29.05.2003 №233).

© Copyright Testing Board.


Krok 2 Medicine 2014 1

1. Head of a department and a trade-union A. Schede-type vertical suspension


group have appealed to the head of a hospital B. Closed reduction
about dismissal of the senior nurse who has C. Intramedullary osteosynthesis
17-year record of service. The facts of charge D. Use of Ilizarov apparatus
were confirmed and recognized by the nurse E. Periosteal osteosynthesis
herself. The nurse lives with a daughter (who is
5. What juice should be included in a complex
divorced and unemployed) and a 9-month-old drug and dietary therapy for patients with
grandson. Make an administrative decision: gastric ulcer or duodenal ulcer and increased
A. To continue the worker in office with a gastric juice acidity in order to accelerate the
warning of dismissal in case of repeated violati- ulcer healing?
on of labor discipline A. Potato, potato and carrot
B. To discharge the worker, i.e. to satisfy B. Apple, birch and apple
demands of the collective C. Pumpkin
C. To issue the sick leave D. Cabbage, cabbage and carrot
D. To embark other officials or public organi- E. Celery, parsley
zations with this problem
E. - 6. A 7-year-old child complains of itchi-
ng, papular erythematous rash, dry skin.
2. A 50-year-old female patient complains of Objectively: there is lichenification in the
aching pain in the lower abdomen. She has popliteal fossae and antecubital spaces.
a history of normal menstrual cycle. At the What immunologic indicator if found in the
age of 40, the patient underwent a surgery for blood serum will verify the diagnosis (atopic
gastric ulcer. Examination findings: abdomen
is soft, in the hypogastrium there is a well- dermatitis)?
defined nodular tumor of limited mobility. A. Total IgE
Vaginal examination findings: the cervix is B. Secretory IgA
clean, of cylindrical shape. Body of the uterus C. IgM
cannot be palpated separately. On both sides D. IgG
of the uterus palpation reveals tight tumors wi- E. IgD
th an uneven surface. The tumors are immobi-
le andl fill the whole pelvic cavity. What is the 7. A 7-year-old patient presents with body
most likely diagnosis? temperature rise up to 39o C , dry cough, pain
in the lateral abdomen. Objectively: there is
A. Krukenberg tumor cyanosis of the nasolabial triangle, inspiratory
B. Ovarian fibroid dyspnea with accessory muscle recruitment.
C. Ovarian granulosa cell tumor Percussion reveals pulmonary dullness; among
D. Bilateral pioovarium auscultation findings there are diminished
E. Subserous metrofibroma breath sounds, crepitant rales. Respiratory
3. A 38-year-old male patient complains of rate is of 50/min, HR- 120/min. Evaluate the
marked dyspnea that escalates with physical grade of respiratory failure in the patient:
exertion. The problems, namely acute chest A. II
pain on the left and cough, arose unexpectedly B. I
2 hours before at work. The pain abated, C. III
but there were progressing dyspnea, dizzi- D. IV
ness, pallor, cold sweat, cyanosis. Auscultation E. 0
reveals the absence of vesicular breath sounds,
radiograph shows a shadow on the left. What 8. A 43-year-old alcohol abuser had not
pathology can be suspected? consumed alcohol for the last two days. In
the evening he claimed to see rats and feel
A. Left-sided spontaneous pneumothorax like they bite his feet. The patient is disori-
B. Pulmonary infarction ented, agitated, all the time attempts to run
C. Pleurisy somewhere. Specify the psychopathological
D. Left-sided pneumonia syndrome:
E. Lung abscess
A. Delirious
4. A 2-year-old boy has been admitted to B. Amential
the casualty department for the pain and C. Oneiroid
deformity of his right thigh. Radiograph D. Choreatic
shows a femoral fracture with longitudinal E. Ganser’s syndrome
displacement. What method of treatment is
indicated for the patient? 9. A 67-year-old male patient complains of
rash, severe pain in the subscapular regi-
on on the right. Objectively: skin in the ri-
ght subscapular region is covered with li-
nearly arranged pink-red edematous lesions
that are somewhat infiltrated, and have clear
Krok 2 Medicine 2014 2

boundaries. On the lesion surface there are of intense pain in the right hypochondrium
vesicles with transparent exudate. What is the irradiating to the right supraclavicular region.
most likely diagnosis? Skin and sclerae are icteric. There is tension
and tenderness in the right hypochondrium
A. Herpes zoster on palpation. Body temperature is 38, 8o C .
B. Duhring dermatitis Blood test results: WBC- 11, 2 · 109 /l, total bi-
C. Erysipelas lirubin - 112 mmol/l (conjugated - 86 mmol/l,
D. Atopic dermatitis unconjugated - 26 mmol/l). What is the most
E. Impetigo likely diagnosis?
10. An 8-year-old boy was brought to the A. Cholangitis
admission department by his parents. Parents B. Acute pancreatitis
report that he has had pain in the right knee C. Acute appendicitis
for the last 9 months, recently mother has noti- D. Pancreatic tumor
ced some limitation of motion in his right leg, E. Perforated duodenal ulcer
and morning stiffness that doesn’t last till the
evening. What is the most likely diagnosis? 15. A 3-year-old girl has had an increase
in body temperature up to 38, 5o C for four
A. Juvenile rheumatoid arthritis days. The child refuses to eat. Over the last
B. Rheumatism two days, nose and mouth breathing has
C. Osteomyelitis of the knee joint become difficult. Mesopharyngoscopy reveals
D. Reactive arthritis hyperthermia and enlargement of tonsils, as
E. Traumatic arthritis well as hyperemia and bulging of the posteri-
11. A 13-year-old girl was admitted to the or wall of the oropharynx, which significantly
gynecology department for having a signifi- narrows the oropharyngeal lumen. What
cant bleeding from the genital tract for 10 complication of quinsy occurred in the pati-
days. The patient has a history of irregular ent?
menstrual cycle since menarche. Menarche A. Retropharyngeal abscess
occurred at the age of 11. Recto-abdominal B. Paratonsillar abscess
examination revealed no pathology. What is C. Parapharyngeal abscess
the provisional diagnosis? D. Phlegmon of the mouth floor
A. Juvenile uterine bleeding E. Laryngostenosis
B. Adenomyosis 16. Within a year, in a maternity hospital there
C. Injury of the external genitalia were 616 livebirths, one stillbirth, one infant
D. Werlhof’s disease died on the 5th day of life. What index can
E. Endometrial polyp most accurately describe this situation?
12. A 64-year-old male patient has a 35- A. Perinatal mortality
year history of chronic pancreatitis. In B. Total mortality
the last 5 years, he claims to observe the C. Birthrate
pain abatement, bloating, frequent bowel D. Infant mortality
movements up to 3-4 times a day, grayish, E. Natural increase
glossy stool with undigested food rests, the
progressive loss of body weight. Change of 17. In one of the surgical departments the
symptoms in the patient is due to overlay of: quality assurance testing of sterilization of
surgical instruments was performed. After
A. Exocrine pancreatic insufficiency an instrument had been treated with 1%
B. Endocrine pancreatic insufficiency phenolphthalein, the solution turned pink.
C. Lactase deficiency syndrome This indicates that the instrument has:
D. Irritable bowel syndrome
E. Chronic enterocolitis A. Synthetic detergent residues
B. Residual blood
13. During the doctor’s round, a 56-year-old C. Drugs residues
male patient with decompensated cirrhosis D. Residual tissue
complains of dizziness, palpitations, moving E. Disinfectant residues
black specks seen before the eyes, general
weakness. The patient is pale, Ps- 110/min, 18. A 21-year-old female patient consulted a
AP- 90/50 mm Hg. What complication is most gynecologist about itching, burning, watery
likely to have occurred in the patient? vaginal discharges with a fish-like smell.
Speculum examination revealed that the cervi-
A. Bleeding from esophageal varices cal and vaginal mucosa was of a normal pi-
B. Hepatocellular insufficiency nk color. Vaginal examination revealed no
C. Hepatic encephalopathy alterations of the uterus and appendages.
D. Acute coronary syndrome Gram-stained smears included clue cells.
E. Paroxysmal tachycardia What is the most likely pathology?
14. A 57-year-old female patient complains
Krok 2 Medicine 2014 3

A. Bacterial vaginosis (gardnerellosis) mitive judgments, is unabe to perform si-


B. Chlamydiosis mple arithmetic operations or explain simple
C. Gonorrhea metaphors. The patient is untidy, takes no
D. Trichomoniasis interest in anything, passive. Considers hi-
E. Candidiasis mself to be completely healthy. Qualify mental
condition of the patient:
19. A 9-year-old patient has measles. On
the 6th day after the rash appeared, the boy A. Total dementia
developed a condition manifested by dyspnea, B. Lacunar (dysmnestic) dementia
barking cough, stenotic respiration. Objecti- C. Somnolentia
vely: the rash on the face, neck and torso D. Korsakoff’s (amnesic) syndrome
turned brown. There is a branny desquamati- E. Hysterical pseudodementia
on. Respiratory rate is 22/min. What compli-
cation should be diagnosed? 23. A 13-year-old boy with hypertrophic cardi-
omyopathy complains of dyspnea on mini-
A. Laryngotracheitis mal exertion. EhoCG reveals asymmetric left
B. Bronchitis ventricular hypertrophy, signs of pulmonary
C. Pneumonia hypertension, dilatation of the left atrium. EF
D. Pharyngitis is 64%. The revealed alterations are indicative
E. Quinsy of:
20. A 26-year-old secundipara at 40 weeks of A. Diastolic heart failure
gestation arrived at the maternity ward after B. Systolic heart failure
the beginning of labor activity. 2 hours before, C. Primary pulmonary hypertension
bursting of waters occurred. The fetus was D. Primary arterial hypertension
in a longitudinal lie with cephalic presentati- E. Symptomatic arterial hypertension
on. Abdominal circumference was 100 cm,
fundal height - 42 cm. Contractions occurred 24. Study of natural illumination for a
every 4-5 minutes and lasted 25 seconds each. workplace in a secondary school classroom
Internal obstetric examination revealed cervi- revealed that the angle of sunlight incidence
cal effacement, opening by 4 cm. Fetal bladder was 25o , window opening angle - 3o , window-
was absent. Fetal head was pressed against the to-floor area ratio - 1:4, daylight ratio - 0,5%,
pelvic inlet. What complication arose in chi- depth ratio - 2. What indicators do not meet
ldbirth? hygienic standards?
A. Early amniorrhea A. Daylight ratio
B. Primary uterine inertia B. Window opening angle
C. Secondary uterine inertia C. Window-to-floor area ratio
D. Discoordinated labor D. Depth ratio
E. Clinically narrow pelvis E. Angle of incidence
21. Examination of a 35-year-old patient wi- 25. Bakers at bread production work in condi-
th rheumatism revealed that the right heart tions of high temperature and high heat radi-
border was 1 cm displaced outwards from the ation. What is used to increase the body’s
right parasternal line, the upper border was resistance to the unfavorable effects of these
on the level with inferior margin of the 1st harmful work environment factors?
rib, the left border was 1 cm in from the left
midclavicular line. Auscultation revealed atri- A. Vitamin preparations
al fibrillation, loud apical first sound, diastolic B. Milk
shock above the pulmonary artery. Echocardi- C. Pectin
ocopy revealed abnormal pattern of the mitral D. Therapeutic and preventive diet № 1
valve motion. What heart disease is characteri- E. Therapeutic and preventive diet number № 3
zed by these symptoms?
26. A 24-year-old male patient got a puncture
A. Mitral stenosis injury below the Poupart’s ligament, which
B. Mitral valve prolapse was accompanied by intense arterial bleedi-
C. Mitral valve insufficiency ng. The best method to temporarily stop the
D. Aortic stenosis bleeding in the patient would be:
E. Tricuspid valve insufficiency
A. Compression band
22. A 54-year-old male patient works as B. Esmarch’s tourniquet
an engineer. At the age of 35, he got C. Maximum limb bending
infected with syphilis and treated it with D. Compressing a blood vessel with a clamp
"traditional remedies". About 5 years ago, E. Wound suturing
he became forgetful, unable to cope with
work, told cynical jokes, bought useless thi- 27. 5 days before, a 26-year-old female patient
ngs, collected cigarette butts in the street. developed an acute condition. Objectively:
Objectively: the patient is indifferent, has marked headache, vomiting, weakness, poor
slow speech, dysarthria, can make only pri- appetite, temperature up to 39o C . Objecti-
Krok 2 Medicine 2014 4

vely: the patient is in a moderately grave agnosed with hidradenitis. What is the most
condition, excited. The face is hyperemic, likely causative agent of this disease?
sclerae are injected. The tongue is coated with
brown fur. The trunk and limbs are covered A. Staphylococci
with plentiful roseolous and petechial rash. B. Streptococci
Hepatosplenomegaly is present. Complement C. Proteus vulgaris
binding reaction with Rickettsia prowazekii D. Pseudomonas aeruginosa
is positive with the titer of 1:640. What drug E. Mixed infection
should be administered?
32. A 36-year-old female has a 7-year history of
A. Doxycycline pollen allergy. Over the last 2 years in August
B. Chloramphenicol and September (during ragweed flowering),
C. Penicillin the patient has had 2-3 asthma attacks that
D. Streptomycin could be treated with one dose of salbutamol.
E. Metronidazole Objectively: body temperature - 36, 5o C ,
respiratory rate - 18/min, Ps- 78/min, AP-
28. A 39-year-old female patient complains of 115/70 mm Hg. There is vesicular breathing
dyspnea when walking, palpitation, edemata above the lungs. Cardiac sounds are sonorous,
in the evening. The patient’s height is 164 cm, of regular rhythm. What drug would be most
weight - 104 kg. Objectively: overnutrition. effective to prevent asthma attacks during the
Heart sounds are weak, and tachycardia is critical season for the patient?
present. The menstrual cycle is not broken.
Blood sugar is 5,6 mmol/l, ACTH-response A. Intalum inhalation
tests revealed no alterations. X-ray of the B. Berotec inhalation
Turkish saddle revealed no pathology. What C. Atrovent inhalation
disease is it? D. Suprastin administration
E. Theopecum administration
A. Alimentary obesity
B. Climax 33. A study of the structure of death causes
C. Pituitary obesity in the urban population revealed that cardi-
D. Diabetes mellitus ovascular diseases accounted for 55,0% of all
E. Cushing’s syndrome (primary hypercorti- deaths. What statistic value represents these
solism) data?

29. A 26-year-old male patient complains of a A. Extensive index


rash on the upper lip skin, which arose on B. Intensive index
a background of influenza with high-grade C. Index of evidence
fever and is accompanied by pain and burning. D. Index of correlation
The rash has been present for 3 days. Objecti- E. Correspondence index
vely: the skin of the upper lip is edematic and
erythematous, grouped vesicles are filled with 34. A 25-year-old female patient complains of
serous fluid and have a rough surface. What is marked weakness, sleepiness, blackouts, di-
the most likely diagnosis? zziness, taste disorder. The patient has a hi-
story of menorrhagia. Objectively: the pati-
A. Herpetic vesicular dermatitis ent has marked weakness, pale skin, cracks in
B. Eczema the corners of mouth, peeling nails, systolic
C. Contact dermatitis apical murmur. Blood test results: RBC -
D. Dermatitis herpetiformis 3, 4 · 1012 /l, Hb- 70 g/l, color index - 0,75,
E. Erythema multiforme platelets - 140 · 109 /l, WBC- 6, 2 · 109 /l. What is
the most likely diagnosis?
30. A 6-year-old boy complains of paroxysmal
pain that occurs after a mental stress, consumi- A. Chronic posthemorrhagic anemia
ng cold drinks or ice cream. After clinical and B. Acute leukemia
instrumental examination the boy has been di- C. Acute posthemorrhagic anemia
agnosed with hypertensive biliary dyskinesia. D. B12 -deficiency anemia
The drugs of the following groups should be E. Werlhof’s disease
administered in the first place:
35. A 51-year-old female is a weaving factory
A. Antispasmodics and choleretics worker with 15 years of service record. During
B. Choleretics and cholekinetics a regular preventive examination she complai-
C. Sedatives and cholekinetics ned of frequent headaches, poor sleep, tingli-
D. Antioxidants ng in the heart, irritability, rapid fatigabili-
E. Antibiotics ty, hearing impairment. For years, the noise
level has exceeded the maximum allowable
31. A 15-year-old patient consulted a concentration by 10-15 dB. A year ago, the
dermatologist about a painful lump in patient underwent a course of treatment for
the armpit. Objectively: there is a walnut- essential hypertension. Specify the most likely
sized node, lymphadenitis, infiltration of the diagnosis:
surrounding tissues. The patient has been di-
Krok 2 Medicine 2014 5

A. Noise disease
B. Essential hypertension A. Hemolytic anemia
C. Neurasthenia B. Iron-deficiency anemia
D. Asthenic-vegetative syndrome C. Protein-deficiency anemia
E. Arteriosclerotic encephalopathy D. B12 -deficiency anemia
E. Hereditary elliptocytosis
36. A 49-year-old male patient who had been
scheduled for a surgery for gastric cancer 40. A lumbar puncture was performed for a
underwent preoperative infusion therapy. Up newborn suspected of having an intracranial
to 3,0 liters of liquid was introduced into the birth injury. Bloody cerebrospinal fluid was
right cubital vein. The following day, he felt a obtained. What hemorrhage occurred in this
dragging pain in the right shoulder. Objecti- case?
vely: on the inner surface of the shoulder
there is a longitudinal hyperemic zone, edema A. Subarachnoid
of skin, a tender cord. What complication B. Cephalohematoma
occurred in the patient? C. Epidural
D. Supratentorial
A. Acute thrombophlebitis E. Subtentorial
B. Venepuncture and edema of paravenous
tissue 41. A 36-year-old female patient complains
C. Paravenous tissue necrosis of intense pain in the knee joints and neck.
D. Acute lymphangitis In the morning she experiences pain in the
E. Paravenous tissue phlegmon interscapular region and leg joints; pain subsi-
des after warm-up gymnastics. The patient is
37. Some of the population of a city distri- overnourished, there is a clicking sound in the
ct have uneven teeth color. The individuals knees when squatting, the knees are somewhat
have white spots, transverse brown stripes on disfigured, painful on palpation. Blood test
the incisors. Occurrence of these symptoms is results: ESR- 18 mm/h, WBC- 8, 0 · 109 /l. Radi-
associated with the quality of drinking water ography reveals subchondral sclerosis in the
from a deep well. Which of the following left knee. What is the basis of this pathology?
components of water can be the cause of the
disease? A. Degenerative processes in cartilage
B. Autoimmune process in the synovium
A. F C. Deposition of urates (tophi) in the articular
B. Ca tissues
C. Mg D. Beta-haemolytic streptococcus
D. J E. Hemarthrosis
E. Fe
42. A 35-year-old female reports heart pain
38. A 22-year-old female patient has been (aching and drilling) occurring mainly in the
delivered by an ambulance team to a surgi- morning in autumn and spring and irradi-
cal clinic with symptoms of acute intestinal ating to the neck, back and abdomen; rapid
obstruction. It is known from the past hi- heartbeat; low vitality. Occurrence of this
story that 2 years ago she was operated for condition is not associated with physical acti-
acute destructive appendicitis. For two years, vity. In the evening, the patient’s condition
she has repeatedly complained of bloating improves. Study of somatic and neurological
and abdominal pain. Which of the followi- status, and ECG reveal no pathology. What
ng etiological factors has led to the intestinal pathology is most likely to have caused these
obstruction in the patient? clinical presentations?
A. Abdominal adhesions A. Somatization depression
B. Ileal diverticulum B. Resting stenocardia
C. Dolichosigma C. Pseudoneurotic schizophrenia
D. Diet violation D. Neurocirculatory asthenia
E. Helminthiasis E. Hypochondriacal depression
39. Mother of a 10-month-old baby reports 43. An hour before an elective surgery, a 56-
significant pallor, poor appetite, enlarged year-old patient of the surgical department
abdomen in the baby. As a neonate, the child got a dramatic increase in blood pressure,
underwent treatment in the in-patient hospi- tachycardia, hand tremor. The patient is
tal for jaundice and anemia. Objectively: the confused, anxious, depressed, fearful, is pessi-
skin is pale and jaundiced, teeth are absent, mistic about the operation outcome, refuses
abdomen is enlarged, spleen is palpable. the surgery. What tactics should be chosen by
Blood test results: Hb- 90 g/l, RBC- 3, 0·1012 /l, a surgeon?
color index - 0,9, microspherocytosis, reti-
culocytosis up to 20%, serum bilirubin - 37
mmol/l, unconjugated bilirubin - 28 mmol/l.
What type of anemia has occurred in the pati-
ent?
Krok 2 Medicine 2014 6

A. Start the surgery after correction of blood has been a smoker since childhood. Objecti-
pressure vely: to - 37, 4o C , respiratory rate is 26/min,
B. Isolate the patient Ps- 82/min, rhythmic. AP- 130/85 mm Hg.
C. Predict the psychological state of the patient There is limited breathing movement in the
D. Organize monitoring of the patient by right side of chest cavity, as well as percussive
medical personnel and mental health counselor dullness and diminished breath sounds. Radi-
E. Organize monitoring of the patient by his ograph shows a homogeneous opacity of the
family members pulmonary field on the right with the mediasti-
num displacement to the affected side. What
44. A 33-year-old female complains of escalati- is the most likely diagnosis?
ng spastic pain in the abdomen after the
psycho-emotional stress. The patient has A. Central lung cancer
intermittent bowel movements, that is 2-3 B. Pleural effusion
bowel movements after waking up alternate C. Pleuropneumonia
with constipation lasting for 1-2 days. Objecti- D. Pulmonary tuberculosis
vely: body weight is unchanged, there is E. Bronchiectasis
moderate pain on palpation of the sigmoid
colon. Hb- 130 g/l, WBC- 5, 2 · 109 /l, ESR- 9 48. A 33-year-old male patient developed a
mm/h. Proctosigmoidoscopy causes pain due condition that had a stormy clinical course:
to spastic bowel condition, intestinal mucosa chills, fever up to 39o C , vomiting, epigastric
is not changed. In the lumen there is a lot of pain, diarrhea with watery smelly feces. 6
mucus. What is the most likely diagnosis? hours before, he ate a raw egg, fried potatoes
with stewed meat, drank some juice. What
A. Irritable bowel syndrome pathogen is likely to have caused this conditi-
B. Crohn’s disease on?
C. Non-specific ulcerative colitis
D. Acute bowel ischemia A. Salmonella
E. Malabsorption syndrome B. Colibacillus
C. Campylobacter
45. An infant is 2 days old. He was born D. Shigella
full-term with signs of intrauterine infection, E. Vibrio cholerae
and therefore receives antibiotics. Neonates
should be given antibiotics at longer intervals 49. Chief physician of a polyclinic encharged
and lower doses compared to older children a district doctor with a task to determine the
and adults because: pathological prevalence of disease N in his
district. What document allows to estimate
A. Neonates have lower glomerular filtration the disease prevalence in the population of a
B. Neonates have lower concentration of medical district?
protein and albumin in blood
C. Neonates have a reduced activity of A. Prophylactic examinations register
glucuronyl transferase B. Statistic coupons (+)
D. Neonates have a decreased blood pH C. Statistic coupons (-)
E. Neonates have higher hematocrit D. Statistic coupons (+) and (-)
E. Vouchers for medical appointments
46. After a holiday in the Crimea, a 36-year-
old female patient presents with severe pain 50. A 76-year-old male consulted a therapi-
in the elbow joints, dyspnea and weakness. st about slow discharge of urine with a small
The body temperature is of 37, 6o C , the skin jet. The patient reported no cardiac problems.
is pale, there is erythema of cheeks and nose, Examination revealed atrial fibrillation with
lower lip ulceration. Visual inspection reveals a heart rate of 72/min and without pulse defi-
no changes in the joints, the right elbow cit. There are no signs of heart failure. ECG
movement is limited. There is murmur and confirms the presence of atrial fibrillation.
pleural friction in the lungs below the ri- From history we know that the arrhythmia
ght angle of the scapula. Cardiac sounds are was detected three years ago. What tactics for
muffled, there is tachycardia, gallop rhythm, the treatment of atrial fibrillation in the pati-
Ps- 114/min. AP- 100/60. What is the most li- ent should be chosen?
kely diagnosis?
A. Does not require treatment
A. SLE B. Digoxin
B. Rheumatic heart disease C. Verapamil
C. Rheumatoid arthritis D. Obzidan
D. Infectious allergic myocarditis E. Ajmaline
E. Dry pleurisy
51. A 53-year-old male has been admitted to
47. A 63-year-old male patient complains of a hospital for an attack of renal colic whi-
cough with expectoration of mucous blood- ch has repeatedly occurred throughout the
streaked sputum, asthma, low-grade fever, year. Objectively: in the region of auricles
general weakness. These presentations have and the right elbow some nodules can be
been observed for 3 months. The patient seen that are covered with thin shiny skin.
Krok 2 Medicine 2014 7

Ps- 88/min, AP- 170/100 mm Hg. There is bi- since he was 16, abuses alcohol, has a history
lateral costovertebral angle tenderness (posi- of CHD. The left lower extremity is colder
tive Pasternatsky’s symptom). The patient than the right one, the skin of extremities is
has been scheduled for examination. What dry, pedal pulse cannot be palpated, femoral
laboratory value would be most helpful for pulse is preserved. What is the most likely di-
making a diagnosis? agnosis?

A. Uric acid A. Obliterating endarteritis


B. Rheumatoid factor B. Diabetic angiopathy
C. ESR C. Leriche syndrome
D. Urine sediment D. Raynaud’s disease
E. Lactic acid E. Deep thrombophlebitis

52. A 58-year-old female patient complai- 56. A 38-year-old male complains of tonic
ns of spontaneous bruises, weakness, bleedi- tension of the masticatory muscles, so that
ng gums, dizziness. Objectively: the mucous he cannot open his mouth. 12 days before, he
membranes and skin are pale with numerous was bitten by an unknown dog. Objectively:
hemorrhages of various time of origin. Lymph there is pronounced tension and twitching of
nodes are not enlarged. Ps- 100/min, AP- the masticatory muscles. What is the most li-
110/70 mm Hg. There are no changes of kely diagnosis?
internal organs. Blood test results: RBC -
A. Tetanus
3,0·1012 /l, Нb - 92 g/l, colour index - 0,9, B. Rabies
anisocytosis, poikilocytosis, WBC - 10·109 /l, C. Hysteria
eosinophils - 2%, stab neutrophils - 12%, D. Trigeminal neuralgia
segmented neutrophils - 68%, lymphocytes E. Apyretic tetanus
- 11%, monocytes - 7%, ESR - 12 mm/h. What
laboratory test is to be determined next for 57. A 72-year-old male had had a moderate
making a diagnosis? headache. Two days later, he developed the
progressing speech disorders and weakness in
A. Platelets the right extremities. The patient has a history
B. Reticulocytes of myocardial infarction, arrhythmia. Study
C. Clotting time of the neurologic status revealed elements of
D. Osmotic resistance of erythrocytes motor aphasia, central paresis of the VII and
E. Fibrinogen XII cranial nerves on the right, central hemi-
paresis on the same side and hyperaesthesia.
53. A 48-year-old male in-patient undergoes What is the most likely diagnosis?
treatment for essential hypertension of II-
B stage. It is known from history that he A. Ischemic stroke
works in a design engineering office. His job B. Hemorrhagic stroke
involves neuro-emotional stress. Which of C. Transient ischemic attack
these foodstuffs do not stimulate the central D. Epidural hematoma
nervous system and can be recommended for E. Brain tumor
the patient?
58. A 28-year-old female patient has been
A. Whole milk admitted to the gynecology department for
B. Meat broths abdominal pain, spotting before and after
C. Mushroom broths menstruation for 5 days. The disease is associ-
D. Vegetable broths ated with the abortion which she had 2 years
E. Carbonated beverages ago. Anti-inflammatory treatment had no
effect. Bimanual examination findings: the
54. In an urban settlement situated on the ri- uterus is enlarged, tight, painful, smooth.
verbank, an outbreak of hepatitis A was regi- Hysteroscopy reveals dark red holes in the
stered. The disease might have water origin. fundus with dark blood coming out of them.
This assumption can be confirmed by growth What diagnosis can be made on the grounds
of the following values of water quality: of these clinical presentations?
A. Number of coli-phages A. Inner endometriosis
B. Escherichia coli index B. Polymenorrhea
C. Oxidability C. Hypermenorrhea
D. Presence of benign leptospirosis pathogen D. Submucous fibromatous node
E. Index of fecal coli-forms E. Dysfunctional uterine bleeding
55. A 48-year-old male patient complains of 59. 2 weeks after having quinsy, a 26-
pain in the lower extremities, especially when year-old male patient got facial edemata,
walking, intermittent claudication, numbness moderate pain in the sacrum. Objectively:
in the fingers, cold extremities, inability to body temperature is 37, 5o C , AP- 100/80 mm
walk more than 100 meters. Sleeps with his Hg. Urinalysis results: RBC- up to 100 fresh
leg lowered. The patient has been a smoker cells in per HPF, protein - 2,2 g/l, hyaline cyli-
Krok 2 Medicine 2014 8

nders - up to 10 per HPF, relative density - the heart’s need for oxygen without aggravati-
1002. What is the most likely diagnosis? ng the disease?
A. Acute glomerulonephritis A. Isosorbide dinitrate
B. Nephroma B. Corinfar
C. Acute pyelonephritis C. Atenolol
D. Urolithiasis D. Streptokinase
E. Chronic glomerulonephritis E. Aminophylline
60. Examination of a full-term 6-day-old infant 64. A 5-year-old girl has had thirst, polyuria,
revealed that different areas of skin had increased appetite for two months. At the
erythemas, flaccid bubbles, eroded surface, same time, there is a 3 kg decrease in body
cracks, peeling of the epidermis looking like weight. During the last week, these presentati-
being scalded with boiling water. There was ons got accompanied by nocturnal enuresis.
positive Nikolsky’s symptom. General condi- Examination revealed hyperglycemia at the
tion of the child was serious. The child was rate of 14 mmol/l. The child has been di-
restless, hypersensitive, febrile. What is the agnosed with type I diabetes. What is the most
most likely diagnosis in this case? likely genesis of this disease?
A. Ritter’s exfoliative dermatitis A. Autoimmune
B. Neonatal phlegmon B. Viral
C. Finger’s pseudofurunculosis C. Bacterial
D. Neonatal pemphigus D. Neurogenic
E. Epidermolysis E. Viral and bacterial
61. A 39-year-old female patient complains of 65. An 8-year-old child with a 3-year history
rapid fatigability, drowsiness, dry skin, hair of diabetes was hospitalized in hyperglycemic
loss, swelling of the face. A month ago, she coma. Specify the initial dose of insulin to be
underwent a surgery for thyrotoxicosis. The administered:
patient has the following gland dysfunction:
A. 0,1-0,2 U/kg of body weight per hour
A. Thyroid (hypothyroidism), due to B. 0,05 U/kg of body weight per hour
inadequate operative technique C. 0,2-0,3 U/kg of body weight per hour
B. Pituitary, due to a tumor D. 0,3-0,4 U/kg of body weight per hour
C. Adrenal E. 0,4-0,5 U/kg of body weight per hour
D. Parathyroid, due to the gland removal
during surgery 66. A 12-year-old girl undergoes regular
E. Ovarian, due to a tumor gastroenterological check-ups for duodenal
ulcer, biliary dyskinesia. What is the
62. A 27-year-old patient has a severe recommended frequency of anti-relapse
headache, nausea and vomiting. Objecti- treatment?
vely: body temperature is 38, 9o C , there is
a haemorrhagic stellate rash on the legs. A. Twice a year
The patient takes meningeal pose in bed. B. Every two months
Meningeal symptoms are strongly positive. C. Every 3 months
Deep reflexes are brisk, uniform. Pathologi- D. Once a year
cal reflexes are absent. It has been suspected E. Three times a year
that the patient has epidemic cerebrospinal
meningitis. Which of additional tests should 67. On the 2nd day of disease a 27-year-old
be performed in the first place to verify the patient complains of unbearable headache,
diagnosis? repeated vomiting. Objectively: the patient
is in a grave condition. He is conscious but
A. Lumbar puncture adynamic. Lies in a forced position with his
B. Echoencephalography head thrown back. There is no skin rash.
C. Rheoencephalography Nuchal muscles are evidently rigid, there are
D. Electroencephalography Kernig’s and Brudzinski’s signs. to - 39, 5o C ,
E. Survey craniogram Ps- 120/min, AP- 130/80 mm Hg. The leading
syndrome of this disease is caused by:
63. 3 hours before, a 68-year-old male pati-
ent got a searing chest pain radiating to the A. Liquor hypertension
neck and left forearm, escalating dyspnea. Ni- B. Liquor hypotension
troglycerin failed to relieve pain but somewhat C. Affection of the cranial nerve nuclei
reduced dyspnea. Objectively: there is cri- D. Haemorrhages in the adrenal glands
mson cyanosis of face. Respiratory rate is E. Hyperthermy
28/min. The patient has vesicular breathing
with isolated sibilant rales. Heart sounds are 68. Two years ago, a 46-year-old patient was
muffled, with a gallop rhythm. Ps- 100/min, diagnosed with stage I silicosis. Currently
AP- 100/65 mm Hg. ECG shows negative T- the patient complains of escalating dyspnea,
wave in V 2 − V 6 leads. What drug can reduce pain in the infrascapular regions. Radiograph
Krok 2 Medicine 2014 9

shows a diffuse enhancement and distorti-


on of lung markings, as well as multiple A. Rheumatoid arthritis
nodular shadows 2-4 mm in diameter. There is B. Osteoarthritis
interlobar pleural density on the right. Dense C. Gout
shadows are found in the hilar regions. Specify D. Pseudogout
the form of radiographic pulmonary fibrosis in E. Multiple myeloma
this case:
73. A 30-year-old female patient has been
A. Nodular delivered to a hospital for sudden dyspnea
B. Interstitial progessing to asthma, sensation of having a
C. Interstitial nodular "lump in the throat", hand tremor, fear of
D. Nodal death. The attack has developed for the first
E. Tumor-like time and is associated with a strong emotion.
There is no previous history. Objectvely: respi-
69. A 63-year-old female complains of general ratory rate - 28/min, Ps- 104/min, rhythmic,
weakness, a feeling of heaviness, compressi- AP- 150/85 mm Hg. The patient has rapid
on in the epigastrium, postprandial fullness, superficial vesicular breathing with extended
nausea, belching after meals. These symptoms expiration. Percussion findings: heart borders
have been observed for about 15 years. are not changed. Cardiac sounds are loud,
Objectively: body temperature is 36, 4o C , rhythmic. What is the most likely diagnosis?
respiratory rate - 20/min, Ps - 88/min, blood
pressure - 115/75 mm Hg. Skin and mucous A. Neurocirculatory asthenia
membranes are pale. Blood test results: RBC - B. Bronchial asthma
2,0·1012 /l, Hb - 100 g/l. Tests revealed parietal- C. Hypertensive crisis
cell antibodies. What is the most likely reason D. Cardiac asthma
for the development of anemia in this patient? E. Thyrotoxic crisis

A. Production of antibodies to intrinsic factor 74. A 42-year-old male patient wth essential
B. Disruption of hemoglobin synthesis hypertension presents with headache, palpi-
C. Disruption of erythropoietin synthesis tations, unexplained fear. Objectively: Ps-
D. Impaired iron absorption 100/min, AP- 200/100 mm Hg, the left border
E. Increased loss of iron of cardiac dullness is displaced by 1,5 cm to
the left, vesicular breathing is present. ECG
70. During dressing of a poorly-granulating shows sinus tachycardia, signs of left ventri-
wound Pseudomonas aeruginosa infection was cular hypertrophy. What drug should be admi-
revealed. What medication would be optimal nistered as an emergency?
for the wound d-bridement?
A. Obzidan
A. Boric acid solution B. Dibazol
B. Biogenic stimulators C. Reserpine
C. Sulfonamides D. Magnesium sulfate
D. Salicylic acid E. Furosemide
E. Antibiotics
75. A 37-year-old male patient has a histrory
71. A 39-year-old male patient complains of of diabetes of moderate severity. On the left
moderate pain and weakness in the shoulder, side of face the patient has a carbuncle. What
back and pelvic girdle muscles, that has been severe complication might have occurred in
progressing for the last 3 weeks; great di- the patient?
fficulty in getting out of bed, going up and
down the stairs, shaving. Dermatomyositis has A. Cavernous sinus thrombosis
been suspected. Blood test results: Hb- 114 g/l, B. Lymphangitis
C. Endarteritis
WBC- 10, 8 · 109 /l, eosinophils - 9%, ESR -22 D. Thrombophlebitis
mm/h, C-reactive protein - (++). The alterati- E. Thromboembolism
on in the following laboratory value wil be of
decisive diagnostic significance: 76. A 42-year-old female patient complains of
a dull pain in her left side, low-grade fever,
A. Creatine phosphokinase accelerated painful urination in small porti-
B. Ceruloplasmin ons. These presentations have been observed
C. Sialic acids for three years. For a long time, the pati-
D. dsDNA antibodies ent has had cystitis with frequent exacerbati-
E. Gamma-globulins ons, there is pulmonary tuberculosis in the
72. A 60-year-old female patient complains of past history. Urinalysis results: microscopic
recurrent pain in the proximal interphalangeal hematuria, leukocyturia. What is the most li-
and wrist joints, their periodic swelling and kely provisional diagnosis?
reddening that have been observed for 4 years.
X-ray picture represents changes in form of
osteoporosis, joint space narrowing and single
usuras. What is the most likely diagnosis?
Krok 2 Medicine 2014 10

A. Renal tuberculosis before the onset of the disease the patient had
B. Urolithiasis examined a dead calf. What is the most likely
C. Chronic pyelonephritis diagnosis?
D. Renal tumor
E. Chronic cystitis A. Cutaneous anthrax
B. Bubonic plague
77. A woman at 30 weeks pregnant has had C. Carbuncle
an attack of eclampsia at home. On admissi- D. Lymphocutaneous tularemia
on to the maternity ward AP- 150/100 mm E. Erysipelas
Hg. Predicted fetal weight is 1500 g. There
is face and shin pastosity. Urine potein is 81. An employee of a petrol station with 15
0, 66o /oo . Parturient canal is not ready for deli- years of service record having contact wi-
very. An intensive complex therapy has been th ethylated gasoline presents with memory
started. What is the correct tactics of this case impairment, bradycardia, sensation of havi-
management? ng a hair in the mouth, skin paresthesia. In
this case, one can assume intoxication with the
A. Delivery by cesarean section following substance:
B. Continue therapy and prolong pregnancy
for 1-2 weeks A. Tetraethyl lead
C. Continue therapy and prolong pregnancy B. Lead chloride
for 3-4 weeks C. Organophosphates
D. Labor induction by intravenous oxytocin or D. Benzene
prostaglandins E. Nitrobenzene
E. Treat preeclampsia and achieve the delivery
by way of conservative management 82. A 38-year-old female suddenly developed
acute inflammatory rash in form of roseolas,
78. Examination of a dead man who died papules, vesicles that are scattered on the skin
from hanging revealed that cadaver spots di- of trunk in irregular and predominantly focal
sappeared when pressed upon and restored manner. The rash appeared a few hours after
after 50 seconds, rigor mortis was moderately visiting a restaurant. The patient complains of
expressed only in the masticatory muscles itching skin. What is the most likely diagnosis?
and the muscles of neck and fingers. Body
temperature was 31o C . Specify the time of A. Toxicodermatosis
death: B. Atopic dermatitis
C. Contact dermatitis
A. 6-7 hours D. Eczema
B. 1-2 hours E. -
C. 16-24 hours
D. 8-10 hours 83. The institutions which take part in medi-
E. 10-18 hours cal examinations can be prevention and
treatment facilities, medical board of Mini-
79. A 65-year-old male patient complains of stry of Defense, medical board of Ministry of
dyspnea that is getting worse with exerti- Home Affairs, medico-social expert commi-
on, morning cough with expectoration of ssions, forensic medical boards etc. What insti-
mucous sputum. For about 15 years, he has tutions are responsible for temporary disabili-
been subject to regular medical check-up ty examination?
for chronic bronchitis. The patient takes
berodual (16 inhaled doses per day). Objecti- A. Prevention and treatment facilities
vely: body temperature is 36, 8o C , RR- 24/min, B. Sanitary-and-prophylactic institutions
Ps- 110/min, AP- 145/90 mm Hg. Auscultati- C. Medico-social expert commissions
on reveals a lot of dry rales above the lungs. D. Medical boards of the Ministry of Defense
FEV1- 65%. What is the optimal tactics of E. Medical boards of the Ministry of Home
further management of the patient? Affairs

A. To administer inhalation corticosteroids 84. After lifting a load, a 36-year-old male


B. To administer antibiotics patient has experienced a severe pain in the
C. To administer theophylline lumbar region, which spread to the right
D. To increase the daily dose of berodual leg and was getting worse when he moved
E. To include short-acting β2 -agonists in the his foot or coughed. Objectively: the long
therapy back muscles on the right are strained. Achi-
lles jerk is reduced on the right. There is a
80. A 49-year-old countryman got an itching pronounced tenderness of paravertebral poi-
papule on the dorsum of his right hand. In nts in the lumbar region. The straight leg raise
the centre there is a vesicle with serosangi- (Lasegue’s sign) is positive on the right. What
nous exudate. Within the next 2 days the additional tests should be performed in the fi-
patient developed a painless edema of hand rst place?
and forearm. On the 4th day the temperature
rose to 38, 5o C , in the right axillary region a
large painful lymph node was found. One day
Krok 2 Medicine 2014 11

A. Radiography of the spinal column A. Central-unit


B. Computed tomography B. Centralized
C. Magnetic resonance tomography C. Decentralized
D. Electromyography D. Free
E. Lumbar puncture E. Combined
85. A 17-year-old male patient consulted a 89. A 43-year-old female patient complai-
therapist about malaise, chills, runny nose, ns of dyspnea, swelling of legs, abdomen
aching muscles and joints, nausea and di- enlargement, pricking heart pain. She has a
arrhea. The patient asks to prescribe him history of tuberculous bronchadenitis, qui-
a lot of painkillers and sedatives (tramadol nsies. The patient’s condition deteriorated 6
or solpadein that help the best, and di- months ago. Objectively: cyanosis, bulging
azepam). Pharyngeal mucosa is pale pink, neck veins, vesicular breathing. Heart borders
clean. Auscultation reveals vesicular breathi- are not displaced. Heart sounds are muffled,
ng. Tachycardia is present. The pupils are di- Ps- 106/min, liver is +4 cm, ascites is present.
lated, there is sluggish response to light. There Low voltage on the ECG has been revealed.
are injection marks on the forearm skin. Duri- Radiograph shows a thin layer of calcium
ng examination, the patient’s manner is vulgar, deposits along the left contour of heart. What
irritable, rude and untruthful. Make a di- treatment should be recommended to the pati-
agnosis: ent?

A. Opioid addiction A. Treatment by a cardiac surgeon


B. Painkillers addiction B. Digitalis preparations
C. Sedative drug addiction C. Anti-TB drugs
D. Acute respiratory disease D. Diuretics
E. Food-born toxic infection E. Vasodilators, nitrates

86. A 4-year-old boy had untimely vacci- 90. A 26-year-old female patient has an 11-
nation. He complains of painful swallowing, year history of rheumatism. Four years ago
headache, inertness, fever. Objectively: the she suffered 2 rheumatic attacks. Over the last
child is pale, has enlarged anterior cervical 6 months there have been paroxysms of atri-
lymph nodes, swollen tonsils with cyanotic al fibrillation every 2-3 months. What option
hyperemia, tonsils are covered with gray-white of antiarrhythmic therapy or tactics should be
pellicles which cannot be easily removed. proposed?
When the pellicles are forcibly removed, the
tonsils bleed. What is the most likely di- A. Prophylactic administration of cordarone
agnosis? B. Immediate hospitalization
C. Defibrillation
A. Oropharyngeal diphtheria D. Lidocaine administration
B. Lacunar tonsillitis E. Heparin administration
C. Pseudomembranous tonsillitis
D. Infectious mononucleosis 91. Blood typing resulted in positive
E. Follicular tonsillitis isohemagglutination reaction with standard
sera of A(II) and B(III) groups and negative
87. Public nurseries are designed as a single or reaction with sera of 0(I) and AB(IV) groups.
several one- or two-storey buildings linked What is this result indicative of?
by covered walkways. Planning the structure
of preschool instiutions is based upon the A. Faulty standard sera
following principle: B. The first blood group
C. The second blood group
A. Principle of group isolation D. The third blood group
B. Principle of autonomy E. The fourth blood group
C. Principle of age distribution
D. Principle of sex distribution 92. A 9-year-old girl has been admitted to
E. Principle of age-sex distribution a hospital for an elevated body temperature
(39, 8o C ), painful dry cough, abdominal pain
88. A city somatic hospital with 300 beds has a on the right. Examination reveals dullness on
main building which houses the therapeutic percussion on the right, diminished breath
and surgical departments. Several separate sounds, crepitus. What study is required to
buildings house the maternity, pediatric and make a diagnosis?
radiologic departments that are connected to
the main building by underground walkways A. Radiography of the chest cavity
and above-ground covered skybridges. Speci- B. USI of the chest cavity
fy the building system of the hospital: C. Pleural puncture
D. Bronchoscopy
E. Bronhography
93. A newborn has purulent discharges from
the umbilical wound, the skin around the navel
Krok 2 Medicine 2014 12

is swollen. The baby’s skin is pale, with a is 0,586 mmol/l, plasma potassium - 7,2 mmol/l.
yellow-gray tint, generalized hemorrhagic rash What treatment is necessary for this patient?
is present. What is the most likely diagnosis?
A. Hemodialysis
A. Sepsis B. Large doses of verospiron
B. Hemorrhagic disease of the newborn C. Plasma volume expanders
C. Hemolytic disease of the newborn D. Glucocorticosteroids
D. Thrombocytopathy E. Heparin
E. Omphalitis
98. A 10-year-old child with a history of
94. A pregnant 26-year-old woman was admi- nonrheumatic carditis has periodic attacks
tted to a hospital for abdominal pain and manifested by heart pain, dyspnea, pallor, hi-
bleeding from the genital tract. Bimanual gh blood pressure, a dramatic increase in heart
examination revealed that uterus was the si- rate up to 180/min. What drug would be most
ze of 9 weeks of pregnancy, the cervical canal effective to treat this patient?
let a finger through. Fetal tissues could be
palpated in the orifice. There was moderate A. Obsidan
vaginal bleeding. What is the tactics of choice? B. Procainamide
C. Lidocaine
A. Instrumental extraction of fetal tissue D. Verapamil
B. Surveillance E. Ajmaline
C. Administration of hormones
D. Hemostatic and antianemic therapy 99. A 45-year-old male patient with acute
E. Therapy for the maintenance of pregnancy abscess of the left lung has suddenly
developed acute chest pain and dyspnea whi-
95. On the second day of the disease a 22- le coughing, tachycardia has increased. The
year-old male patient complains of high-grade control Ro-gram shows left lung collapse, the
fever, headache in the region of forehead and air in the left pleural cavity and a horizontal
superciliary arches, and during eye movement; fluid level. What is the mechanism of this
aching muscles and joints. Objectively: body complication?
temperature is 39o C . Face is hyperemic,
sclerae are injected. The mucous membrane A. Abscess burst into the pleural cavity
of the soft palate and posterior pharyngeal B. Bullae rupture of the left lung
wall is bright hyperemic and has petechial C. Inflammation spread to the visceral pleura
hemorrhages. What changes in the hemogram D. Atelectasis of the left lung
are typical for this disease? E. Acute cardiovascular insufficiency

A. Leukopenia 100. A 24-year-old male patient had been


B. Leukocytosis diagnosed with class III diffuse toxic goi-
C. Neutrocytosis ter. There is moderate hyperthyroidism. A
D. Anemia surgery was suggested, and the patient agreed
E. Accelerated ESR to it. What preoperative measures should be
taken for prevention of thyrotoxic crisis in the
96. A 44-year-old male patient complains of postoperative period?
severe non-localized abdominal pain, pain in
the right shoulder girdle, repeated vomiting, A. Administration of antithyroid drugs
red urine. The onset of the disease is associ- B. Minimally invasive surgical techniques
ated with alcohol consumption. The face is C. Bed rest
hyperemic. AP- 70/40 mm Hg. Abdominal D. Detoxification therapy
radiography reveals no pathological shadows. E. Administration of corticosteroids
Hemodiastase is 54 mg/h/l. Prothrombin is
46%. What is the provisional diagnosis? 101. A 26-year-old male patient complains of
pain in the right knee, which is getting worse in
A. Acute pancreatitis the morning. Two weeks before, he consulted
B. Acute myocardial infarction an urologist about prostatitis. Objectively:
C. Perforated gastric ulcer conjunctivitis is present. There is also peri-
D. Thrombosis of mesenteric vessels articular edema of the knee joint, redness of
E. Aneurysm of the abdominal aorta the overlying skin. Rheumatoid factor was not
detected. Until further diagnosis is specified,
97. A 41-year-old male patient was delivered it would be reasonable to start treatment with
to a hospital unconscious. During the previ- the following antibiotic:
ous 7 days he had been taking large doses
of biseptolum for a cold. The night before, A. Tetracyclines
he began complaining of dyspnea, especially B. Cephalosporins
when lying down, swollen legs, 2-day urinary C. Penicillins
retention. In the morning he had seizures and D. Aminoglycosides
lost consciousness. Objctively: noisy breathi- E. Lincosamides
ng at the rate of 30/min, edematous legs and
lumbar region, Ps- 50/min. Plasma creatinine 102. A 66-year-old female patient has been
Krok 2 Medicine 2014 13

admitted to a hospital for massive gross A. Furosemide


hematuria with release of shapeless blood B. Mannitol
clots, frequent painful urination. The pati- C. Hydrochlorthiazide
ent also reports a moderate weight loss wi- D. Spironolactone
thin 3-4 months. Gross hematuria that was E. Moduretic
not accompanied by pain and dysuria first
occurred three months ago for no apparent 106. For 3 days, a 28-year-old emale patient
reason, and after a few days the bleeding had had the body temperature increase up to
subsided independently. What is the most li- 38o C , weakness, poor appetite, nausea, a si-
kely diagnosis? ngle vomiting. On the 4th day the temperature
was normal, the condition improved, but the
A. Bladder tumor jaundice developed. Objectively: moderate
B. Urolithiasis ictericity of skin, +3 cm enlarged liver of
C. Renal tumor elastic consistency. Ortner’s, Kehr’s and
D. Chronic cystitis Voznesensky’s symptoms are negative. What
E. Acute cystitis test will verify the diagnosis?

103. A 36-year-old female pesented to a A. IgM Anti-HAV detection


gynecological hospital with a significant B. Complete blood count
bleeding from the genital tract and a 1-month C. Ultrasound of the abdomen
delay of menstruation. Bimanual examinati- D. Total bilirubin
on revealed soft barrel-shaped cervix. Uterus E. AST activity
was of normal size, somewhat softened.
Appendages were unremarkable on both si- 107. An 11-year-old girl has been immunized
des. Speculum examination revealed that the according to her age and in compliance with
cervix was cyanotic, enlarged, with the the the calendar dates. What vaccinations should
external orifice disclosed up to 0,5 cm. Uri- the children receive at this age?
ne hCG test was positive. What is the most A. Diphtheria and tetanus
likely diagnosis? B. TB
A. Cervical pregnancy C. Polio
B. Uterogestation D. Hepatitis B
C. Abortion in progress E. Pertussis
D. Threatened miscarriage 108. A 40-year-old male patient has had heavi-
E. Ectopic pregnancy ness in the epigastric region for the last 6
104. A 47-year-old female patient has an 8- months. He has not undergone any exami-
year history of ulcerative colitis, has been nations. The night before, he abused vodka. In
treated with glucocorticoids. She complai- the morning there was vomiting, and 30 mi-
ns of cramping pain in the umbilical regi- nutes after physical activity the patient experi-
on and left iliac region which has signifi- enced dizziness and profuse hematemesis.
cantly increased during the past 2 weeks, di- What pathology should be suspected in the
arrhea with mucus and blood 4-6 times a day, first place?
elevated body temperature up to 38 − 39o C , A. Mallory-Weis’s syndrome
headache and pain in the knee joints. Objecti- B. Menetrier’s disease
vely: the patient is in moderate condition, C. Gastric ulcer
Ps- 108/min, AP- 90/60 mm Hg; heart and D. Perforated ulcer
lungs are unremarkable; the tongue is moi- E. Zollinger-Ellison syndrome
st; abdominal muscle tone is significantly
decreased; peristaltic noises are absent. What 109. A 55-year-old patient complains of severe
complication developed in the patient? itching, burning and pain in the eyes, skin
redness in the outer corners of the palpebral
A. Toxic dilatation of the colon fissure. Objectively: skin around the outer
B. Perforation of the colon corners of the palpebral fissure is macerated,
C. Enterorrhagia eczematous, there are single moist cracks.
D. Stricture of the colon Palpebral conjunctiva is hyperemic, quaggy.
E. Colon carcinoma There are minor discharges in form of stringi-
105. A 10-year-old child has been admitted ng mucus. What is the most likely diagnosis?
to a hospital with a closed craniocerebral A. Chronic conjunctivitis
injury with a suspected cerebral edema. The B. Acute conjunctivitis
patient is in grave condition, unconscious. C. Sty
The dyspnea, tachycardia, hypertension are D. Blepharitis
present. Muscle tone is increased, there is E. Atopic eyelid dermatitis
nystagmus, pupillary and oculomotor reacti-
ons are impaired. The mandatory component 110. A 3-month-old infant has occipital
of intensive care is dehydration. What diuretic alopecia, restless sleep, excessive sweating.
is adequate in this case? What disease can you think of?
Krok 2 Medicine 2014 14

A. Rickets A. Apply a bandage, give an injection of


B. Spasmophilic diathesis vasodilators
C. Anemia B. Administer heart medications
D. Phosphate diabetes C. Put the feet into hot water
E. Chondrodystrophy D. Rub the feet with snow
E. Apply an alcohol compress
111. A 50-year-old male in a grave conditi-
on has been admitted to the intensive care 116. A 21-year-old male patient got a deep cut
unit. It is known from life history that the pati- wound in his right thigh. In the emergency
ent works in agriculture, and 3 hours ago was room a surgeon on duty performed pri-
engaged into insecticide treatment of crops mary debridement of the wound and pri-
for control of colorado potato beetle. Conditi- mary wound closure with a suture. After 4
on on admission: acrocyanosis, bronchorrhea, days, there appeared pain, redness, edema,
tachypnea, AP- 100/60 mm Hg, Ps- 44/min. purulent discharge from the wound gap, body
What method of efferent therapy would be temperature rose up to 39o C . What kind of
most appropriate at this stage? wound complication can you think of and what
actions should be taken?
A. Hemosorbtion
B. Hemodialysis A. Wound abscess, remove the sutures and
C. Plasmapheresis drain the wound
D. Lymphosorption B. Infiltration, apply a hot compress
E. Plasma dialysis C. Erysipelas, prescribe antibiotics
D. Tetanus, active-passive immunization agai-
112. An 80-year-old patient complains of nst tetanus
constantly urinating small amounts, a feeling E. Lymphangitis, apply a hot compress
of pressure in the lower abdomen. Objectively:
there is a suprapubic spherical bulging with 117. A general practitioner visited a 2-year-old
percussion dullness over it. What syndrome child and diagnosed him with measles. The
occurred in the patient? child attends a nursery, has a 5-year-old si-
ster. What document must be filled in for the
A. Paradoxical ischuria effective antiepidemic measures in the given
B. Urinary incontinence health locality?
C. Dysuria
D. Enuresis A. Emergency notification on infectious di-
E. Pollakisuria sease (form № 058/o)
B. Carer’s leave certificate
113. Carpathian region is characterized by C. Infant’s record (report form № 112/o)
permanently high (over 80%) air humidi- D. House call record (form № 031/o)
ty. In the cold season the population of this E. Sick leave
region feels very cold at moderately low
temperatures. This is due to an increase in 118. An employee has been diseased for 4
the heat transfer by: months, further treatment is necessary, the
patient is incapacitated. Who is authorized to
A. Convection provide further disability examination of this
B. Emission patient?
C. Evaporation
D. Conduction A. Medical and social expert board
E. Radiation B. Medical consultative board
C. Physician in charge and chief of department
114. An 18-year-old girl complains of breast D. Chief physician of a medical facility
pain and engorgement, headaches, irri- E. Deputy chief responsible for disability
tability, swelling of the lower extremities. examination
These symptoms have been observed since
menarche and occur 3-4 days before the 119. It has been suspected that a newborn has
regular menstruation. Gynecological exami- congenital diaphragmatic hernia (asphyctic
nation revealed no pathology. Make a di- incarceration). What study will allow to confi-
agnosis: rm the diagnosis?
A. Premenstrual syndrome A. Plan radiography of the chest cavity
B. Neurasthenia B. Plan abdominal radiography
C. Renal disease C. Irrigography
D. Mastopathy D. Pneumoirrigoscopy
E. Cardiovascular disorder E. Fibroesophagogastroduodenoscopy
115. A 56-year-old male patient has been 120. A 25-year-old female presented to a
delivered to the emergency department wi- women’s welfare clinic and reported the
th frostbite on both feet. What aid should be inability to get pregnant within 3 years of
rendered to the victim? regular sexual activity. Examination revealed
Krok 2 Medicine 2014 15

increased body weight, male pattern of she hears the voice of her brother who tells
pubic hair growth, excessive pilosis of thi- her to go home. The patient is anxious, suspi-
ghs, dense enlarged ovaries, monophasic basal cious, looks around all the time. Specify the
temperature. What is the most likely di- psychopathological syndrome:
agnosis?
A. Hallucinatory
A. Polycystic ovarian syndrome B. Generalized anxiety disorder
B. Adnexitis C. Paranoiac
C. Adrenogenital syndrome D. Paraphrenic
D. Premenstrual syndrome E. Depressive
E. Gonadal dysgenesis
125. During the first home visit to a full-term
121. A factory’s sectorial doctor selects a boy after his discharge from the maternity
group of persons who often fall ill for thorough hospital a pediatrician revealed a symmetri-
monitoring. At the same time he takes into cal swelling of mammae without skin changes
consideration the number of etiologically over them, swelling of the scrotum. The body
related cases with temporary disability in temperature was of 36, 5o C . The baby was
each of the employees over the last year. An calm, sucked the mother’s breast actively.
employee falls into this group if the number of What condition should you think of?
sickness cases is:
A. Hormonal crisis of the newborn
A. 4 or more B. Neonatal mastitis
B. 1 or more C. Sclerema
C. 2 or more D. Necrotic neonatal phlegmon
D. 3 or more E. Congenital adrenal dysfunction
E. 6 or more
126. A 30-year-old male patient complains
122. A 23-year-old female consulted a of inertness, low-grade fever, bleeding gums,
gynecologist on the 20th day postpartum peri- frequent quinsies, aching bones. Objecti-
od about pain in the left breast, purulent vely: the patient has pale skin and mucous
discharge from the nipple. Objectively: Ps- membranes, sternalgia, +2 cm liver, +5 cm
120/min, t - 39 C . The left breast is painful, painless spleen. Blood test results: RBC-
larger than the right one, hyperemic. In the 2, 7 · 1012 /l, Нb- 80 g/l, WBC- 3 · 109 /l,
upper quadrant there is an infiltrate sized eosinophils - 4%, basophils - 5%, blasts -
10x15 cm with a softening inside. Blood test 4segmented neutrophils - 17%, lymphocytes -
results: ESR- 50 mm/h, WBC- 15, 0 · 109 /l. 29%, myelocytes - 25%, promyelocytes - 12%,
What is the tactics of choice? monocytes - 2%, platelets - 80 · 109 /l, ESR -
57 mm/h. What test should be performed to
A. Refer to the surgical department for operati- verify the diagnosis?
ve treatment
B. Refer to the gynecology department A. Sternal puncture
C. Refer to the postpartum department B. Trephine biopsy
D. Refer to a polyclinic surgeon for conservati- C. Lymph node biopsy
ve treatment D. Lumbar puncture
E. Lance the breast abscess in the women’s E. Chest X-ray
health clinic
127. A 46-year-old male patient complains of
123. A family lives in the town situated within periodic epigastric pain that occurs at night.
the zone of radiation pollution. A 6-year-old Objectively: HR- 70/min, AP- 125/75 mm Hg,
child had been ill with ARVI for 19 days. The tenderness in the epigastric region is present.
child was undergoing outpatient treatment EGD confirms duodenal ulcer of 0,6 cm in
and was nursed by his mother, a cafe worker. diameter. Test for H. Pylori is positive. Whi-
Specify the order of disability examination: ch of the given antisecretory drugs will be
a compulsory element of the treatment regi-
A. Sick leave is granted for the entire period of men?
the child’s illness
B. Sick leave is granted for a total of 14 days, A. Omeprazole
and after that period is over, a carer’s leave B. Famotidine
certificate is issued C. Pirenzepine
C. Sick leave is not granted, only a carer’s leave D. Atropine
certificate is issued E. Maalox
D. Sick leave is granted for 14 days, after this
period is over, no document is issued 128. A 47-year-old male patient has been lately
E. Sick leave is granted for 7 days, after this complaining of compressing chest pain that
period is over, a carer’s leave certificate is issued occurs when he walks a distane of 700-800 m.
Once a week, he drinks 2 liters of beer. Rise in
124. A 49-year-old female patient with schi- arterial pressure has been observed for the last
zophrenia is all the time listening to somethi- 7 years. Objectively: Ps- 74/min, AP- 120/80
ng, insists that "there is a phone in her head"as mm Hg. The bicycle ergometry performed at
Krok 2 Medicine 2014 16

workload of 75 watts shows 2 mm ST -segment A. Botulism


depression in V 4 − V 6 leads. What is the most B. Shigellosis
likely diagnosis? C. Salmonellosis
D. Cholera
A. Exertional stenocardia, II functional class E. Yersiniosis
B. Exertional stenocardia, III functional class
C. Exertional stenocardia, IV functional class 133. A full-term neonate weighing 4500 g was
D. Vegetative-vascular dystonia of hypertensi- born asphyxiated with Apgar score of 4-6 poi-
ve type nts. During the delivery shoulder dystocia
E. Alcoholic cardiomyopathy occurred. Neurologic assessment revealed
non-focal neurologic symptoms, total flaccid
129. A puerperant is 28 years old. It’s the 3rd paresis of the upper extremities since the arm
day post-partum after a second, normal, term was atonic and pronated. Grasping, Babki-
delivery. The body temperature is of 36, 8o C , n’s and Moro’s reflexes were absent. What
Ps- 72/min, AP- 120/80 mm Hg. Mammary segments of spinal cord had been affected?
glands are moderately engorged, the nipples
are clean. Abdomen is soft, painless. The A. V - T hI
fundus is 3 fingers’ breadth below the navel. B. I - II
Moderate bloody lochia are present. What di- C. III - IV
agnosis can be made? D. T hI - T hV
E. T hV I - T hV 
A. Physiological course of the postpartum
period 134. A newborn (mother’s I pregnancy) wei-
B. Subinvolution of uterus ghing 3500 g has been found to have jaundice,
C. Postpartum metroendometritis lethargy, reduced reflexes. Objectively: second
D. Remains of placental tissue after childbirth grade jaundice of skin with saffron tint, li-
E. Lactostasis ver - +2 cm, spleen - +1 cm. Urine and feces
are yellow. Blood count: Hb- 100 g/l, RBC-
130. A 29-year-old unconscious patient has 3, 2 · 1012 /l, WBC- 18, 7 · 109 /l, mother’s blood
been delivered to a hospital. Objectively: skin type - 0(I) Rh(+), baby’s blood type - A(II)
and mucous membranes are pale, cyanotic, Rh(-), bilirubin - 170 mmol/l, indirect fraction.
breath sounds are dramatically diminished ALT, AST rates are normal. What is the most
on the right and cannot be auscultated in the likely disease in the child?
lower parts, at the level of the 6 rib along the
anterior axillary line there is a wound hole wi- A. Hemolytic disease of newborn, AB0-conflict
th moderate bleeding and passage of air duri- B. Perinatal hepatitis
ng inspiration. Radiography reveals a bullet in C. Hemolytic disease of newborn, Rh-conflict
the pleural cavity. What is the medical tactics D. Biliary atresia
of choice? E. Physiologic jaundice
A. Emergency thoracotomy 135. As a result of a road accident a 45-
B. Thoracoscopy with removal of bullet year-old male patient got multiple injuries,
C. Converting a tension pneumothorax into a namely closed fractures of the right humerus
simple (open) pneumothorax and the left antebrachial bones with a di-
D. Thoracostomy splacement of bone fragments, a closed blunt
E. Tight bandage on a wound abdominal injury. The patient was delivered
to the emergency department 30 minutes after
131. A patient had four generalized convulsive the injury. Objectively: the skin is pale. AP-
seizures within a day. Between the seizures the 90/20 mm Hg, there is pain and deformati-
patient did not come to waking consciousness on at the fracture sites. Abdomen is tense,
(was in a coma or stupor). Specify his state: palpation causes severe pain, there is rebound
tenderness (positive Blumberg’s sign). What is
A. Status epilepticus the treatment tactics of choice?
B. Frequent generalized seizures
C. Frequent jacksonian seizures A. Urgent diagostic laparotomy
D. Hysterical attacks B. Infusion therapy to stabilize blood pressure
E. Frequent complex partial seizures C. Fracture immobilization, analgesia
D. Local anesthetic blockade of fractures
132. A 12-year-old boy presents with nausea, E. Additional tests to specify the diagnosis
frequent repeated vomiting that first occurred
after eating canned vegetables. Objectively: 136. A 34-year-old male visited Tajikistan.
the patient has dry mucous membranes, After return, he complains of fever up
muscular hypotonia, anisocoria, mydriasis, to 40o C which occurs every second day
dysphagia and dysarthria. What is the most and is accompanied by chills, sweating.
likely diagnosis? Hepatosplenomegaly is present. Blood test
results: RBC- 3 · 1012 /l, Нb- 80 g/l, WBC- 4 ·
109 /l, eosinophils - 1%, stab neutrophils - 5%,
segmented neutrophils - 60%, lymphocytes -
Krok 2 Medicine 2014 17

24%, monocytes - 10%, ESR - 25 mm/h. What A. Pericardiocentesis and immediate


is the provisional diagnosis? thoracotomy
B. Oxygen inhalation
A. Malaria C. Puncture of the pleural cavity on the left
B. Infectious mononucleosis D. Conservative treatment, infusion of
C. Sepsis adrenomimetics
D. Typhoid fever E. Pleural cavity drainage
E. Leptospirosis
141. A puerperant is 32 years old, it’s her first
137. A 23-year-old male patient consulted childbirth, term precipitate labor, the III peri-
a doctor about pain occurring in the lower od is unremarkable, the uterus is contracted,
third of the thigh with weight bearing acti- tight. Examination of the birth canal revealed
vities and unloaded. The patient denies any a rupture in the left posterior vaginal wall that
injuries to the region. Objectively: the skin was closed with catgut. Two hours later, the
is of normal color, deep palpation reveals patient complained of a feeling of pressure on
pastosity and tenderness, movements of the the anus, pain in the perineum, minor vaginal
knee joint are limited. Radiograph of the di- discharges, edema of the vulva. These clinical
stal femoral metaepiphyseal region shows a presentations are indicative most likely of:
zone of degradation and spicules. In blood:
immature cells are present, there are no signs A. Vaginal hematoma
of inflammation. What is the most likely di- B. Hysterocervicorrhexis
agnosis? C. Hemorrhoids
D. Hysterorrhesis
A. Osteosarcoma E. Hypotonic bleeding
B. Hyperparathyroid dystrophy
C. Chronic osteomyelitis 142. A hospital nutrition unit received a batch
D. Multiple myeloma of beef. Sanitation physician examined the
E. Marble bone disease meat and revealed the presence of 5 bladder
worms per 40 cm2 of meat. Give the hygienic
138. It is planned to organize a rural outpati- assessment of meat:
ent clinic. The patients will be able to visit the
doctors of the following specialities: A. Liable to technical utilization
B. Conditionally admissible
A. Therapeutist, dentist, pediatrician, C. Adulterated
obstetrician-gynecologist D. Good-quality
B. Therapeutist, pediatrician, neurologist E. Poor-quality
C. Pediatrician, obstetrician-gynaecologist,
ophthalmologist 143. A 47-year-old female patient complains of
D. Pediatrician, therapeutist, ophthalmologist having pain on swallowing and difficult passi-
E. Obstetrician-gynaecologist, therapeutist ng of solid food for two months. The pati-
ent has taken to the liquid and semi-liquid
139. A 12-year-old girls has minor functi- food. During the last week the liquid food has
onal and morphological abnormalities: 1,0 D barely passed through. General condition is
myopia, reduced body resistance. The patient satisfactory, the patient is undernourished, the
has no history of chronic diseases. Over the appetite is preserved, there is a fear of eating.
last year, there were 4 cases of respiratory di- What is the provisional diagnosis?
seases. The girl belongs to the following health
group: A. Esophageal carcinoma
B. Esophageal stricture
A. II C. Esophageal foreign body
B. I D. Esophageal varices
C. III E. Esophageal achalasia
D. IV
E. V 144. A 31-year-old female patient complai-
ns of infertility, amenorrhea for 2 years after
140. 4 weeks after a myocardial infarction, the artificial abortion that was complicated by
a 56-year-old male patient developed acute endometritis. Objectively: examination of the
heart pain, pronounced dyspnea. Objectively: external genitalia revals no pathology, there is
the patient’s condition is extremely grave, female pattern of hair distribution. According
there is marked cyanosis of face, swelling and to the functional tests, the patient has biphasic
throbbing of neck veins, peripheral pulse is ovulatory cycle. What form of infertility is the
absent, the carotid artery pulse is rhythmic, case?
130 bpm, AP- 60/20 mm Hg. Auscultation
of heart reveals extremely muffled sounds, A. Uterine
percussion reveals heart border extension in B. Ovarian
both directions. What is the optimal treatment C. Pituitary
tactis for this patient? D. Hypothalamic
E. Immunological
Krok 2 Medicine 2014 18

145. A 49-year-old male patient consulted analyses of the daily urinary excretion of
a doctor about difficult swallowing, voice albumin revealed microalbuminuria (200-
hoarseness, weight loss. These symptoms have 300 mg/day). Glomerular filtration rate
been gradually progressing for the last 3 is 105 ml/min. Blood pressure is within
months. Objectively: the patient is exhausted, normal range. Normalization of the followi-
supraclavicular lymph nodes are enlarged. ng indicator should be the first-priority task
Esophagoscopy revealed no esophageal in the secondary prevention of diabetic
pathology. Which of the following studies is nephropathy:
most appropriate in this case?
A. Glycosylated hemoglobin
A. Computed tomography of chest and medi- B. C-peptide
astinum C. Blood insulin
B. X-ray of lungs D. Fasting glucose
C. Multiplanar imaging of esophagus E. Glycemia 2 hours after a meal
D. Radioisotope investigation of chest
E. Ultrasound investigation of mediastinum 150. 10 days after birth, a newborn developed
a sudden fever up to 38, 1o C . Objectively:
146. A 36-year-old male patient complains of the skin in the region of navel, abdomen
having headache, obstructed nasal breathing, and chest is erythematous; there are multi-
purulent nasal discharge for 2 weeks. A month ple pea-sized blisters with no infiltration at the
before, he had his right maxillary premolar fi- base; single bright red moist erosions with epi-
lled. Radiolography revealed an intense opaci- dermal fragments on the periphery. What is
ty of the right maxillary sinus. Diagnostic your provisional diagnosis?
puncture revealed a large amount of thick
malodorous crumbly pus. What is the most A. Epidemic pemphigus of newborn
likely diagnosis? B. Syphilitic pemphigus
C. Streptococcal impetigo
A. Chronic suppurative odontogenic sinusitis D. Vulgar impetigo
B. Acute purulent sinusitis E. Atopic dermatitis
C. Chronic purulent sinusitis
D. Chronic atrophic sinusitis 151. An emergency physician arrived to provi-
E. Tumor of the maxillary sinus de medical care for a hangman taken out
of the loop by his relatives. The doctor
147. A 5-year-old boy has a history of repeated revealed no pulse in the carotid arteries, lack
pneumonia, frequent acute respiratory viral of consciousness, spontaneous breathing and
diseases. Objectively: exertional dyspnea, mi- corneal reflexes; cadaver spots on the back
nor fatigabilty. There is a systolic murmur and posterior parts of extremities. A person
having its epicenter in the IV intercostal can be declared dead if the following sign is
space on the left. Left relative dullness is present:
found along the midclavicular line. Accordi-
ng to the findings of instrumental methods of A. Cadaver spots
examination (electrocardiography, echocardi- B. Lack of spontaneous breathing
ography), the patient has been diagnosed wi- C. Lack of corneal reflexes
th ventricular septal defect, subcompensation D. Pulselessness
stage. What is the main method of treatment? E. Unconsciousness

A. Operative therapy 152. A 69-year-old male patient has been


B. Phytotherapy hospitalized with hypothermia. Objectively:
C. Does not require treatment the patient is pale, has shallow breathing. AP-
D. Conservative treatment 100/60 mm Hg, Ps- 60/min. Palpation of the
E. Indomethacin abdomen and chest reveals no pathological
signs. The body temperature is of 34, 8o C .
148. A patient with autoimmune thyroi- The patient’s breath smells of alcohol. Give
ditis accompanied by multinodular goiter treatment recommendations:
underwent the right lobe ectomy and subtotal
resection of the left lobe. What drug should A. Warm bath + intravenous administration of
be administered to prevent postoperative warm solutions
hypothyroidism? B. Body warming with hot-water bags
C. Forced diuresis
A. L-thyroxine D. Rubbing with alcohol and snow
B. Merkazolil E. Antishock therapy
C. Iodomarin
D. Lithium drugs 153. A 26-year-old male patient consulted a
E. Insulin doctor abut sore throat, fever up to 38, 2o C .
A week before, the patient had quinsy, di-
149. A 49-year-old female patient has type dn’t follow medical recommendations. On
1 diabetes of moderate severity. The di- examination, the patient had forced position
sease is complicated by retinopathy and of his head, trismus of chewing muscles. Left
polyneuropathy. Besides that, repeated peritonsillar region is markedly hyperemic,
Krok 2 Medicine 2014 19

swollen. What is the provisional diagnosis? upper extremities and decreased pressure
in the lower ones, extension of the left
A. Left-sided peritonsillar abscess heart border, blowing systolic murmur in the
B. Meningitis interscapular region. ECG shows the hori-
C. Phlegmonous tonsillitis zontal axis of heart. Radiography reveals left
D. Pharyngeal diphtheria cardiomegaly, costal usuration. What is the
E. Tonsil tumour most likely diagnosis?
154. A 21-year-old female patient has been A. Aortarctia
hospitalized on an emergency basis because B. Aortic stenosis
of severe dyspnea, pain in the left side of C. Patent ductus arteriosus
chest. Body temperature is 38, 8o C . The condi- D. Ventricular septal defect
tion developed three days ago. Respiratory E. Atrial septal defect
rate is 42/min, auscultation reveals shallow
breathing. There is percussive dullness on 158. During the preventive examinati-
the right starting from the middle of the on a 17-year-old young man reports no
blade, breath sounds cannot be heard. The left health problems. Objectively: the patient is
border of heart is 3 cm displaced outwards. undernourished, asthenic; blood pressure is
Embryocardia is present, HR is 110/min. The 110/70 mm Hg, Ps- 80/min. Heart borders
right hypochondrium is painful on palpation. are within normal range. Auscultation reveals
What urgent therapeutic measures should be three apical heart sounds, murmurs are absent.
taken in this situation? ECG shows no pathological changes, PCG
registers the S3 occurring 0,15 seconds after
A. Emergency puncture of the pleural cavity the S2. How can you interpret these changes?
B. Administration of penicillin antibiotics
C. Injection of Lasix A. Physiologic S3
D. Injection of cardiac glycosides B. Fout-ta-ta-rou (three-component rhythm)
E. Transferring the patient to the thoracic C. Protodiastolic gallop rhythm
surgery department D. Presystolic gallop rhythm
E. Physiologic S4
155. A 77-year-old male patient complains of
inability to urinate, bursting pain above the 159. After having the flu, a 39-year-old
pubis. The patient developed acute condi- male patient with a history of Addison’s
tion 12 hours before. Objectively: full uri- disease developed a condition manifested
nary bladder is palpable above the pubis. by weakness, depression, nausea, vomi-
Rectal prostate is enlarged, dense and elastic, ting, diarrhea, hypoglycemia. AP- 75/50
well-defined, with no nodes. Interlobular mm Hg. Blood test results: low corti-
sulcus is distinct. Ultrasonography results: costerone and cortisol, 13-oxycorticosteroids,
prostate volume is 120 cm3 , it projects into the 17-oxycorticosteroids levels. What condition
bladder cavity, has homogeneous parenchyma. developed in the patient?
Prostate-specific antigen rate is of 5 ng/ml.
What is the most likely disease that caused A. Acute adrenal insufficiency
acute urinary retention? B. Acute gastritis
C. Acute enterocolitis
A. Prostatic hyperplasia D. Collapse
B. Prostate carcinoma E. Diabetes mellitus
C. Sclerosis of the prostate
D. Tuberculosis of the prostate 160. A female patient complains of being
E. Acute prostatitis unable to get pregnant for 5 years. A complete
clinical examination brought the following
156. At the first minute of life a full-term results: hormonal function is not impaired,
infant born with umbilical cord entangled urogenital infection hasn’t been found, on
around his neck has total cyanosis, apnea, hysterosalpingography both tubes were filled
HR- 80/min, hypotonia and areflexia. There with the contrast medium up to the isthmic
are no signs of meconium aspiration. After segment, abdominal contrast was not visuali-
the airway suctioning the newborn did not zed. The patient’s husband is healthy. What
start breathing. What is the next action of the tactics will be most effective?
doctor?
A. In-vitro fertilization
A. ALV with a 100% O2 mask B. Insemination with husband’s sperm
B. Intravenous administration of adrenaline C. ICSI within in-vitro fertilization program
C. Intravenous administration of etamsylate D. Hydrotubation
D. Tracheal intubation and ALV E. Laparoscopic tubal plasty
E. Stimulation of the skin along the spine
161. A 19-year-old primiparous woman wi-
157. Examination of an 11-year-old boy th a body weight of 54,5 kg gave birth at
revealed frequent nosebleeds, fatigue when 38 weeks gestation to a full-term live girl
walking, underdevelopment of the lower half after a normal vaginal delivery. The girl’s wei-
of the body, increased blood pressure in the ght was 2180,0 g, body length - 48 cm. It is
Krok 2 Medicine 2014 20

known from history that the woman has been suspected?


a smoker for 8 years, and kept smoking duri-
ng pregnancy. Pregnancy was complicated by A. Heavy-metal coniosis
moderate vomiting of pregnancy from 9 to 12 B. Silicosis
weeks pregnant, edemata of pregnancy from C. Silicatosis
32 to 38 weeks. What is the most likely cause D. Carbon pneumo coniosis
of low birth weight? E. Bronchitis

A. Fetoplacental insufficiency 166. A 12-year-old boy periodically has short


B. Low weight of the woman episodes (10-15 seconds) of a brief loss of
C. Woman’s age awareness with a dazed look and eyes stare in
D. First trimester preeclampsia an upright position, blank expression of face,
E. Third trimester preeclampsia absence of motions and subsequent amnesia.
Specify the described state:
162. In a cold weather, the emergency room
admitted a patient pulled out of the open A. Absence seizure
water. There was no respiratory contact wi- B. Obnubilation
th the water. The patient is excited, pale, C. Trance
complains of pain, numbness of hands and D. Fugue
feet, cold shiver. Respiratory rate is 22/min, E. Sperrung
AP- 120/90 mm Hg, Ps- 110/min, rectal
temperature is 34, 5o C . What kind of warming 167. Evaluation results of sanitary and hygiene
is indicated for this patient? conditions in a 4-bed ward were as follows:
ward area - 30 m2 , height - 3,2 m, temperature
A. Passive warming - 20o C , humidity - 55%, air velocity - 0,1 m/s,
B. Infusion of 37o C solutions window-to-floor area ratio - 1:5, daylight ratio
C. Hot compresses - 0,6%, concentration of carbon dioxide in the
D. Warm bath air - 0,1%. Which of the given indicators does
E. Hemodialysis with blood warming not meet hygienic requirements?
163. A week before, a 65-year-old male pati- A. Daylight ratio
ent suffered an acute myocardial infarction. B. Air velocity
His general condition has deteriorated: he C. Window-to-floor area ratio
complains of dyspnea at rest, pronounced D. Concentration of carbon dioxide in the air
weakness. Objectively: edema of the lower E. Ward area
extremities, ascites is present. Heart borders
are extended, paradoxical pulse is 2 cm di- 168. A 25-year-old patient works as a tractor
splaced from the apex beat to the left. What is driver. Four days ago, he got pain in the left
the most likely diagnosis? axillary region, general weakness, fever up to
38o C . He hadn’t sought medical helf until a
A. Acute cardiac aneurysm painful solid lump appeared in this region.
B. Recurrent myocardial infarction Objectively: in the left axilla there is a very
C. Acute pericarditis painful cone-shaped mass sized 3x2,5 cm, wi-
D. Cardiosclerotic aneurysm th a destruction in the center of the pointed
E. Pulmonary embolism vertex. The surrounding skin is hyperemic,
there are purulent discharges. What is the
164. A 41-year-old patient cosulted a most likely diagnosis?
dermatologist about discoloration, thickening,
brittleness of toenails. These symptoms have A. Hydradenitis
been present for about five years. Objecti- B. Carbuncle
vely: nail plates in all toes are thickened, of C. Furuncle
dirty yellow color, lustreless, crumble over the D. Lymphadenitis
edge. Microscopy of the nail plate material E. Abscess
treated with alkali revealed mycelial filaments.
Material inoculation onto Sabouraud medium 169. A 19-year-old male patient complains of
resulted in growth of the Trichophyton rubrum intense pain in the left knee joint. Objectively:
colony. What is the most likely diagnosis? the left knee joint is enlarged, the overlyi-
ng skin is hyperemic, the joint is painful on
A. Rubromycosis of toenails palpation. Blood test results: RBC- 3, 8 · 1012 /l,
B. Candidal onychia Hb- 122 g/l, lymphocytes - 7, 4 · 109 /l, platelets
C. Psoriasis of the nails - 183 · 109 /l. ESR- 10 mm/h. Duke bleeding
D. Nail dystrophy time is 4 minutes, Lee-White clotting time - 24
E. Epidermophytosis of nails minutes. A-PTT is 89 sec. Rheumatoid factor
165. Examination of an electric welder with is negative. What is the most likely diagnosis?
15 years of service record revealed dry rales
in the lower lung fields. Radiograph shows
diffuse nodules sized 3-4 mm in the middle
and lower lung fields. What disease can be
Krok 2 Medicine 2014 21

A. Hemophilia, hemarthrosis A. Sorting, medical assistance, evacuation


B. Werlhof’s disease B. Sorting, evacuation, treatment
C. Rheumatoid arthritis C. Medical assistance, evacuation, isolation
D. Thrombocytopathy D. Isolation, rescue activity, recovery
E. Hemorrhagic vasculitis, articular form E. Sorting, recovery, rescue activity
170. A 29-year-old female patient complai- 174. A 36-year-old injured has been taken
ns of dyspnea and palpitations on exertion. to the emergency station with open tibi-
According to her mother, as a child she had al fractures. Examination reveals bleeding:
heart murmur, did not undergo any exami- pulsating blood spurts out of the wound. What
nations. Objectively: the patient has pale skin, medical manipulations are required at this
Ps- 94/min, rhythmic. AP- 120/60 mm Hg. In stage of medical care?
the II intercostal space on the left auscultation
reveals a continuous rasping systolodiastolic A. Apply a tourniquet on the thigh proximal
murmur, diastolic shock above the pulmonary to the source of bleeding and transport the
artery. Blood and urine are unremarkable. patient to the operating room
What is the most likely diagnosis? B. Immobilize the fracture and transport the
patient to the operating room
A. Patent ductus arteriosus C. Apply a tourniquet on the thigh distal to the
B. Atrial septal defect source of bleeding and transport the patient to
C. Ventricular septal defect the operating room
D. Aortarctia D. Stop the bleeding by a compressive bandage
E. Tetralogy of Fallot and transport the patient to the operating room
E. Transport the patient to the operating room
171. A 6-year-old boy had had a quinsy. 9
days later, there appeared edemata of the 175. The outpatient department of a city
face, extremities and trunk, general health hospital works also as a 60-bed day hospital
condition deteriorated. Urine became turbid. for somatic patients. The department operates
Objectively: expressive edemata, ascites. AP- in a single-shift mode. What specialist selects
100/55 mm Hg, diuresis - 0,2 l of urine per patients for admission to the day hospital?
day. Results of the biochemical blood analysis:
total protein - 50 g/l, cholesterol - 11,28 A. District doctor
mmol/l, urea - 7,15 mmol/l, creatinine - 0,08 B. Resident doctor
mmol/l. Urinalysis results: leukocytes - 3-5 per C. Chief of the outpatient department
HPF, red blood cells are absent. What is the D. Deputy chief of the hospital
provisional diagnosis? E. Chief physician of the hospital

A. Acute glomerulonephritis 176. A 67-year-old female patient with


B. Acute pyelonephritis hypertensive crisis has asthma, cough with
C. Urolithiasis expectoration of frothy pink sputum, moist
D. Acute renal failure rales in the lungs. The patient stays in sitti-
E. Chronic glomerulonephritis ng position, respiratory rate is 40/min, AP-
214/136 mm Hg, heart rate - 102/min. What
172. A 13-year-old boy has had abdominal is the most rational tactics of this patient
pain, bloating, nausea, liquid fatty gray stool management?
with putrid smell for the last 3 years. Palpati-
on reveals epigastric tenderness, as well as A. Intravenous administration of furosemide
tenderness in the Desjardins’ pancreatic point, B. Urgent pneumography
Chauffard’s triangle; there is positive Mayo- C. Bed rest, lying position
Robson’s sign. Failure of exocrine pancreatic D. Intravenous administration of a β -blocker
function has been suspected. What is the most E. Tactics can be determined after ECG and
informative method for evaluating the state of chest radiography
exocrine pancreatic function?
177. A 23-year-old primigravida at 39 weeks
A. Fecal elastase-1 determination gestation has been admitted to the maternity
B. Blood serum trypsin determination ward with irregular contractions. The intensi-
C. Sonography of the pancreas ty of uterine contractions is not changing,
D. Blood and urine amylase determination the intervals between them stay long. Bi-
E. Scatological study manual examination reveals that the cervix
is centered, soft, up to 1,5 cm long. There is no
173. Explosion of a tank with benzene at a cervical dilatation. What diagnosis should be
chemical plant has killed and wounded a large made?
number of people. There are over 50 victims
with burns, mechanical injuries and intoxicati-
on. Specify the main elements of medical care
and evacuation of population in this situation:
Krok 2 Medicine 2014 22

A. Pregnancy I, 39 weeks, preliminary period A. Iron-deficiency anemia


B. Pregnancy I, 39 weeks, labor I, 1 period, the B. Acute leukemia
latent phase C. B12 -deficiency anemia
C. Pregnancy I, 39 weeks, labor I, period 1, the D. Vegetative-vascular dystonia
active phase E. Aplastic anemia
D. Pregnancy I, 39 weeks, birth I, 1 period, the
acceleration phase 182. During the ultrasound study of carotid
E. Pregnancy I, 39 weeks, pathological prelimi- and vertebral arteries a 74-year-old pati-
nary period ent developed a condition manifested by di-
zziness, weakness, nausea, transient loss of
178. A 49-year-old male patient complains of consciousness. Objectively: pale skin, AP-
retrosternal pain, heartburn, weight loss of 8 80/60 mm Hg, Ps- 96/min of poor volume.
kg over the last year, constipation, weakness. ECG shows sinus tachycardia, left ventricular
The patient has been a smoker for 20 years, hypertrophy. Focal neurological symptoms
and has a 10-year history of gastroesophageal were not found. What is the provisional di-
reflux disease. The patient is asthenic, has dry agnosis?
skin. EGD revealed an ulcer in the lower
third of the esophagus and esophageal stri- A. Carotid sinus syncope
cture accompanied by edema, hyperemia and B. Orthostatic syncope
multiple erosions of the mucosa. What study C. Morgagni-Adams-Stokes attack
is required for more accurate diagnosis? D. Complete atrioventricular block
E. Acute cerebrovascular accident
A. Biopsy of the esophageal mucosa
B. X-ray examination of the esophagus 183. On the 10th day postpartum a puerperant
C. Respiratory test for Helicobacter pylori woman complains of pain and heaviness in
D. pH-metry of the esophagus and the stomach the left breast. Body temperature is 38, 8o C ,
E. Fecal occult blood test Ps- 94 bpm. The left breast is edematic, the
supero-external quadrant of skin is hyperemic.
179. A 63-year-old male patient with persi- Fluctuation symptom is absent. The nipples di-
stent atrial fibrillation complains of moderate scharge drops of milk when pressed. What is a
dyspnea. Objectively: peripheral edemata are doctor’s further tactics?
absent, vesicular breathing is present, heart
rate - 72/min, AP- 140/90 mm Hg. What combi- A. Antibiotic therapy, immobilization and
nation of drugs will be most effective for the expression of breast milk
secondary prevention of heart failure? B. Compress to both breasts
C. Inhibition of lactation
A. Beta-blockers, ACE inhibitors D. Physiotherapy
B. Beta-blockers, cardiac glycosides E. Opening of the abscess and drainage of the
C. Cardiac glycosides, diuretics breast
D. Cardiac glycosides, ACE inhibitors
E. Diuretics, beta-blockers 184. During the breast self-exam a 37-year-old
female patient revealed a lump in the lower
180. A 57-year-old male patient had an attack inner quadrant of her left breast. Palpation
of retrosternal pain that lasted more than confirms presence of a mobile well-defined
1,5 hours. Objectively: the patient is inert, neoplasm up to 2 cm large. Peripheral lymph
adynamic, has pale skin, cold extremities, poor nodes are not changed. What is the way of
volume pulse, heart rate - 120/min, AP- 70/40 further management?
mm Hg. ECG shows ST elevation in II, III,
aVF leads. What condition are these changes A. Ultrasound examination of breasts,
typical for? mammography, fine-needle aspiration bi-
opsy
A. Cardiogenic shock B. Anti-inflammatory therapy, physiotherapy
B. Arrhythmogenic shock C. Radical mastectomy
C. Perforated gastric ulcer D. Ultrasound monitoring of genitals during
D. Acute pericarditis the entire course of antiestrogens therapy,
E. Acute pancreatitis systemic enzyme therapy, phytotherapy
E. Case follow-up
181. During an exam, a 22-year-old female
student fainted. She grew up in a family with 185. An 8-year-old child was hospitalized
many children, has a history of frequent acute for fever up to 39, 8o C , inertness, moderate
respiratory infections. Objectively: the patient headache, vomiting. Examination revealed
has pale skin and mucous membranes, split- meningeal symptoms. Lumbar puncture was
end hair, brittle nails. Blood test results: RBC- performed. The obtained fluid had raised
2, 7 · 1012 /l, Hb- 75 g/l, color index - 0,7, WBC- opening pressure, it was transparent, with
3, 2 · 109 /l, platelets - 210 · 109 /l, ESR- 30 mm/h. the cell count of 450 cells per 1 mcL (mai-
Blood serum iron is 6 mmol/l. What is the most nly lymphocytes - 90%), glucose level of 2,6
likely diagnosis? mmol/l. What causative agent might have
caused the disease in the child?
Krok 2 Medicine 2014 23

A. Enterovirus
B. Meningococcus A. Chronic adrenal insufficiency
C. Koch’s bacillus B. Diabetes mellitus
D. Staphylococcus C. Coronary artery disease
E. Pneumococcus D. Chronic pancreatitis
E. Pulmonary tuberculosis
186. A 25-year-old female has a self-detected
tumor in the upper outer quadrant of her ri- 190. A 42-year-old male patient has been
ght breast. On palpation there is a painless, delivered to a hospital in a grave condition
firm, mobile lump up to 2 cm in diameter, with dyspnea, cough with expectoration of
peripheral lymph nodes are not changed. In purulent sputum, fever up to 39, 5o C . The
the upper outer quadrant of the right breast first symptoms appeared 3 weeks ago. Two
ultrasound revealed a massive neoplasm with weeks ago, a local therapist diagnosed him wi-
increased echogenicity sized 21x18 mm. What th acute right-sided pneumonia. Over the last
is the most likely diagnosis? 3 days, the patient’s condition deteriorated:
there was a progress of dyspnea, weakness,
A. Fibroadenoma lack of appetite. Chest radiography confirms a
B. Lactocele rounded shadow in the lower lobe of the right
C. Diffuse mastopathy lung with a horizontal fluid level, the right si-
D. Mammary cancer nus is not clearly visualized. What is the most
E. Mastitis likely diagnosis?
187. A 49-year-old female patient complai- A. Abscess of the right lung
ns of itching, burning in the external geni- B. Acute pleuropneumonia
tals, frequent urination. The symptoms have C. Right pulmonary empyema
been present for the last 7 months. The pati- D. Atelectasis of the right lung
ent has irregular menstruation, once every E. Pleural effusion
3-4 months. Over the last two years she has
had hot flashes, sweating, sleep disturbance. 191. An 11-year-old boy complains of general
Examination revealed no pathological weakness, fever up to 38, 2o C , pain and swelli-
changes of the internal reproductive organs. ng of the knee joints, feeling of irregular
Complete blood count and urinalysis showed heartbeat. 3 weeks ago, the child had quinsy.
no pathological changes. Vaginal smear Knee joints are swollen, the overlying skin
contained 20-25 leukocytes per HPF, mixed and skin of the knee region is reddened, local
flora. What is the most likely diagnosis? temperature is increased, movements are li-
mited. Heart sounds are muffled, extrasystole
A. Menopausal syndrome is present, auscultation reveals apical systolic
B. Cystitis murmur that is not conducted to the left ingui-
C. Trichomonas colpitis nal region. ESR is 38 mm/h. CRP is 2+, anti-
D. Vulvitis streptolysin O titre - 400. What is the most
E. Bacterial vaginosis likely diagnosis?
188. A 45-year-old male patient complains of A. Acute rheumatic fever
acute pain in his right side irradiating to the B. Vegetative dysfunction
right thigh and crotch. The patient claims also C. Non-rheumatic carditis
to have frequent urination with urine which D. Juvenile rheumatoid arthritis
resembles a meat slops. The patient has no E. Reactive arthritis
previous history of this condition. There is
costovertebral angle tenderness on the right 192. A 28-year-old male patient complains
(positive Pasternatsky’s symptom). What is of sour regurgitation, cough and heartburn
the most likely diagnosis? that occurs every day after having meals,
when bending forward or lying down. These
A. Urolithiasis problems have been observed for 4 years.
B. Acute appendicitis Objective status and laboratory values are
C. Acute pyelonephritis normal. FEGDS revealed endoesophagitis.
D. Acute cholecystitis. Renal colic What is the leading factor in the development
E. Acute pancreatitis of this disease?
189. After a holiday in the Crimea, a 49- A. Failure of the lower esophageal sphincter
year-old male patient with a history of lung B. Hypersecretion of hydrochloric acid
tuberculosis felt increased weakness, peri- C. Duodeno-gastric reflux
odic dizziness, easing bowel movements with D. Hypergastrinemia
abdominal pain, the need for additional salti- E. Helicobacter pylori infection
ng his meals. The patient has noted that his
condition improves after some sweet tea and 193. On admission a 35-year-old female
validol taken sublingually. Objectively: there reports acute abdominal pain, fever up to
is an intense darkening of skin, AP- 70/50 mm 38, 8o C , mucopurulent discharges. The pati-
Hg, glycemia is 3,0 mmol/l. What is the possi- ent is nulliparous, has a history of 2 artificial
ble cause of health deterioration: abortions. The patient is unmarried, has sexual
Krok 2 Medicine 2014 24

contacts. Gynecological examination reveals scan showed a destruction area in the center
no uterus changes. Appendages are enlarged, of the shadow. Sputum analysis revealed
bilaterally painful. There is profuse purulent MTB. The patient was diagnosed with focal
vaginal discharge. What study is required to tuberculosis. What phases of tuberculosis are
confirm the diagnosis? the identified changes typical for?
A. Bacteriologic and bacteriascopic studies A. Infiltration and disintegration
B. Hysteroscopy B. Infiltration and dissemination
C. Curettage of uterine cavity C. Resorption and scarring
D. Vaginoscopy D. Disintegration and dissemination
E. Laparoscopy E. Calcification and resorption
194. A 20-year-old female consulted a 198. A 43-year-old female complains of si-
gynecologist about not having menstrual gnificant weakness, sore throat, occurrence
period for 7 months. History abstracts: of multiple unexplained bruises on her skin.
early childhood infections and frequent These symptoms have been present for a
tonsillitis, menarche since 13 years, regular week, the disease is associated with quinsy
monthly menstrual cycle of 28 days, painless which she had some time before. Objectively:
menstruation lasts 5-6 days. 7 months ago the body temperature - 38, 9o C , respiratory rate -
patient had an emotional stress. Gynecologi- 24/min, Ps- 110/min, AP- 100/65 mm Hg. The
cal examination revealed no alterations in the patient has pale skin, petechial rash on the
uterus. What is the most likely diagnosis? extremities, enlarged lymph nodes. Blood test
results: Hb- 80 g/l, RBC- 2, 2 · 1012 /l; WBC-
A. Secondary amenorrhea
B. Primary amenorrhea 3, 5 · 109 /l; blasts - 52%; eosinophils - 2%;
C. Algomenorrhea stab neutrophils - 3%; segmented neutrophils
D. Spanomenorrhea - 19%; lymphocytes - 13%; monocytes - 1%;
E. Cryptomenorrhea platelets - 35 · 109 /l. ESR - 47 mm/h. What test
is required to specify the diagnosis?
195. A 48-year-old female has been admi-
tted to the gynecology department for pain in A. Immunophenotyping
the lower right abdomen and low back pain, B. Protein electrophoresis
constipations. Bimanual examination findings: C. Lymph node biopsy
the uterus is immobile, the size of a 10-week D. Determination of anti-platelet antibody
pregnancy, has uneven surface. Aspirate from titer
the uterine cavity contains atypical cells. What E. Cytogenetic study
diagnosis can be made?
199. A 47-year-old male patient complains of
A. Hysterocarcinoma compressive chest pain that occurs both at rest
B. Cervical cancer and during light physical activity; irregular
C. Metrofibroma heartbeat. These problems arose 3 months
D. Colon cancer ago. The patient’s brother died suddenly at the
E. Chorionepithelioma age of 30. Objectively: Ps- 84/min, arrhythmic,
AP- 130/80 mm Hg. ECG confirms signs of
196. A 27-year-old male patient consulted left ventricular hypertrophy, abnormal Q-
a doctor about pain in the lower third waves in V 4 − V 6 leads. EchoCG reveals that
of the thigh with weight bearing activiti- interventricular septum is 1,7 cm, left ventri-
es and unloaded. Two years ago, the pati- cular wall thickness is 1,2 cm. What is the most
ent underwent treatment in the casualty likely diagnosis?
depatment for the open fracture of the lower
third of femur. The fracture healed slowly, the A. Hypertrophic cardiomyopathy
healing process was accompanied by prulent B. Neurocirculatory asthenia
inflammation. Objectively: edema of the lower C. Exertional angina
third of the thigh, elevated local temperature. D. Myocarditis
Radiograph shows signs of destruction and E. Pericarditis
sequestration. What is the most likely di-
agnosis? 200. Within the structure of the region’s
population the share of persons aged 0 to 14
A. Chronic post-traumatic osteomyelitis years is 25%, the share of persons aged 50
B. Osteosarcoma years and older is 30%. What concept most
C. Tuberculosis of femur accurately describes this demographic situati-
D. Hematogenous osteomyelitis on?
E. Multiple myeloma
A. Regressive type of population age structure
197. A 30-year-old male patient had been B. Progressive type of population age structure
admitted to the TB hospital because of the C. Cohort reproduction
following changes detected by fluorography: D. Stationary type of population age structure
an ill-defined shadow of low intensity up to 1 E. Immigration of population
cm in diameter in the S1 of the right lung. CT
INSTRUCTIONAL BOOK
Testing Board

TEST ITEMS FOR LICENSING EXAMINATION: KROK 2. MEDICINE.

Kyiv. Testing Board.


(English language).

Approved to print 25.03/№61. Paper size 60х84 1/8


Offset paper. Typeface. Times New Roman Cyr. Offset print.
Conditional print pages 24. Accounting publishing pages 28.
Issue. 1440 copies
List of abbreviations

A/G Albumin/globulin ratio


A-ANON Alcoholics anonymous
ACT Abdominal computed tomography
ALT Alanin aminotranspherase
AP Arterial (blood) pressure
AST Aspartat aminotranspherase
BP Blood (arterial) pressure
BR Breathing rate
bpm Beats per minute
C.I. Color Index
CBC Complete blood count
CHF Chronic heart failure
CT Computer tomography
DIC Disseminated intravascular coagulation
DCC Doctoral controlling committee
DM-2 Non-Insulin dependent diabetes mellitus
DTP Anti diphtheria-tetanus vaccine
ECG Electrocardiogram
ESR Erythrocyte sedimentation rate
FC Function class
FEGDS Fibro-esphago-gastro-duodenoscopy
Gy Gray
GIT Gastrointestinal tract
Hb Hemoglobin
HbA1c Glycosylated hemoglobin
Hct, Ht Hematocrit
HDL High-density lipoproteins
HR Heart rate
IDDM Insulin dependent diabetes mellitus
IHD Ischemic heart disease
IU International unit
MSEC Medical and sanitary expert committee
NIDDM Non-Insulin dependent diabetes mellitus
pCO2 CO2 partial pressure
pO2 O2 partial pressure
Pm Per minute
Ps Pulse rate
R Roentgen
RBC Red blood count
Rh Rhesus
RR Respiratory rate
S1 (S1) Heart sound 1
S2 (S2) Heart sound 2
TU Tuberculin unit
U Unit
USI Ultrasound investigation
V/f Vision field
WBC White blood count
X-ray Roentgenogram

Vous aimerez peut-être aussi